CINXE.COM

What kind of wall is this and how can I repair it? - Home Improvement Stack Exchange

<!DOCTYPE html> <html itemscope itemtype="https://schema.org/QAPage" class="html__responsive " lang="en"> <head><script type="text/javascript" src="/_static/js/bundle-playback.js?v=HxkREWBo" charset="utf-8"></script> <script type="text/javascript" src="/_static/js/wombat.js?v=txqj7nKC" charset="utf-8"></script> <script>window.RufflePlayer=window.RufflePlayer||{};window.RufflePlayer.config={"autoplay":"on","unmuteOverlay":"hidden"};</script> <script type="text/javascript" src="/_static/js/ruffle/ruffle.js"></script> <script type="text/javascript"> __wm.init("https://web.archive.org/web"); __wm.wombat("https://diy.stackexchange.com/questions/268276/what-kind-of-wall-is-this-and-how-can-i-repair-it","20230306075114","https://web.archive.org/","web","/_static/", "1678089074"); </script> <link rel="stylesheet" type="text/css" href="/_static/css/banner-styles.css?v=S1zqJCYt" /> <link rel="stylesheet" type="text/css" href="/_static/css/iconochive.css?v=3PDvdIFv" /> <!-- End Wayback Rewrite JS Include --> <title>What kind of wall is this and how can I repair it? - Home Improvement Stack Exchange</title> <link rel="shortcut icon" href="https://web.archive.org/web/20230306075114im_/https://cdn.sstatic.net/Sites/diy/Img/favicon.ico?v=491b13b20e11"> <link rel="apple-touch-icon" href="https://web.archive.org/web/20230306075114im_/https://cdn.sstatic.net/Sites/diy/Img/apple-touch-icon.png?v=26d138b3a1da"> <link rel="image_src" href="https://web.archive.org/web/20230306075114im_/https://cdn.sstatic.net/Sites/diy/Img/apple-touch-icon.png?v=26d138b3a1da"> <link rel="search" type="application/opensearchdescription+xml" title="Home Improvement Stack Exchange" href="/web/20230306075114/https://diy.stackexchange.com/opensearch.xml"> <link rel="canonical" href="https://web.archive.org/web/20230306075114/https://diy.stackexchange.com/questions/268276/what-kind-of-wall-is-this-and-how-can-i-repair-it"/> <meta name="viewport" content="width=device-width, height=device-height, initial-scale=1.0, minimum-scale=1.0"> <meta property="og:type" content="website"/> <meta property="og:url" content="https://web.archive.org/web/20230306075114/https://diy.stackexchange.com/questions/268276/what-kind-of-wall-is-this-and-how-can-i-repair-it"/> <meta property="og:site_name" content="Home Improvement Stack Exchange"/> <meta property="og:image" itemprop="image primaryImageOfPage" content="https://web.archive.org/web/20230306075114im_/https://cdn.sstatic.net/Sites/diy/Img/apple-touch-icon@2.png?v=13c04ccda0b9"/> <meta name="twitter:card" content="summary"/> <meta name="twitter:domain" content="diy.stackexchange.com"/> <meta name="twitter:site" content="@StackDIY"/> <meta name="twitter:creator" content="@StackDIY"/> <meta name="twitter:title" property="og:title" itemprop="name" content="What kind of wall is this and how can I repair it?"/> <meta name="twitter:description" property="og:description" itemprop="description" content="Album here. In the master bedroom of my house, the front wall has some surface cracks that I'm trying to address before repainting. For some background info, this is an early 1900s rowhome. The int..."/> <script id="webpack-public-path" type="text/uri-list">https://cdn.sstatic.net/</script> <script src="https://web.archive.org/web/20230306075114js_/https://ajax.googleapis.com/ajax/libs/jquery/1.12.4/jquery.min.js"></script> <script defer src="https://web.archive.org/web/20230306075114js_/https://cdn.sstatic.net/Js/third-party/npm/@stackoverflow/stacks/dist/js/stacks.min.js?v=1d4d417772d8"></script> <script src="https://web.archive.org/web/20230306075114js_/https://cdn.sstatic.net/Js/stub.en.js?v=e56950123f65"></script> <link rel="stylesheet" type="text/css" href="https://web.archive.org/web/20230306075114cs_/https://cdn.sstatic.net/Shared/stacks.css?v=56e23d88c162"> <link rel="stylesheet" type="text/css" href="https://web.archive.org/web/20230306075114cs_/https://cdn.sstatic.net/Sites/diy/primary.css?v=1dbda87fa603"> <link rel="alternate" type="application/atom+xml" title="Feed for question 'What kind of wall is this and how can I repair it?'" href="/web/20230306075114/https://diy.stackexchange.com/feeds/question/268276"> <script> StackExchange.ready(function () { StackExchange.using("postValidation", function () { StackExchange.postValidation.initOnBlurAndSubmit($('#post-form'), 2, 'answer'); }); StackExchange.question.init({showAnswerHelp:true,totalCommentCount:0,shownCommentCount:0,enableTables:true,questionId:268276}); styleCode(); StackExchange.realtime.subscribeToQuestion('73', '268276'); StackExchange.using("gps", function () { StackExchange.gps.trackOutboundClicks('#content', '.js-post-body'); }); }); </script> <link rel="stylesheet" type="text/css" href="https://web.archive.org/web/20230306075114cs_/https://cdn.sstatic.net/Shared/Channels/channels.css?v=a5fae8812988"> <script> StackExchange.ready(function () { StackExchange.realtime.init('wss://qa.sockets.stackexchange.com'); StackExchange.realtime.subscribeToReputationNotifications('73'); StackExchange.realtime.subscribeToTopBarNotifications('73'); }); </script> <script type="application/json" data-role="module-args" data-module-name="Shared/options.mod">{"options":{"locale":"en","serverTime":1678089074,"routeName":"Questions/Show","stackAuthUrl":"https://web.archive.org/web/20230306075114/https://stackauth.com","networkMetaHostname":"meta.stackexchange.com","site":{"name":"Home Improvement Stack Exchange","description":"Q\u0026A for contractors and serious DIYers","isNoticesTabEnabled":true,"enableNewTagCreationWarning":false,"insertSpaceAfterNameTabCompletion":false,"id":73,"cookieDomain":".stackexchange.com","childUrl":"https://web.archive.org/web/20230306075114/https://diy.meta.stackexchange.com","negativeVoteScoreFloor":null,"enableSocialMediaInSharePopup":true,"protocol":"https"},"user":{"fkey":"2c18411b777ea03baea08fb46c0b7df9d44c6584e5748f81e49d7acfb2f8f921","tid":"190c8054-d8ce-c45d-b9d2-0d99e667e404","rep":0,"isAnonymous":true,"isAnonymousNetworkWide":true},"realtime":{"newest":true,"active":true,"tagged":true,"staleDisconnectIntervalInHours":0},"events":{"postType":{"question":1},"postEditionSection":{"title":1,"body":2,"tags":3}}}}</script> <script type="application/json" data-role="module-args" data-module-name="Shared/settings.mod">{"settings":{"userMessaging":{},"tags":{},"subscriptions":{"defaultBasicMaxTrueUpSeats":250,"defaultFreemiumMaxTrueUpSeats":50,"defaultMaxTrueUpSeats":1000},"snippets":{"renderDomain":"stacksnippets.net"},"site":{"allowImageUploads":true,"enableImgurHttps":true,"enableUserHovercards":true,"forceHttpsImages":true,"stacksEditorPreviewEnabled":true},"questions":{"enableQuestionTitleLengthLiveWarning":true,"enableSavesFeature":true,"maxTitleSize":150,"questionTitleLengthStartLiveWarningChars":50},"intercom":{"appId":"inf0secd","hostBaseUrl":"https://web.archive.org/web/20230306075114/https://stacksnippets.net"},"paths":{"jQueryUICSSPath":"https://web.archive.org/web/20230306075114/https://ajax.googleapis.com/ajax/libs/jqueryui/1.12.0/themes/smoothness/jquery-ui.css","jQueryUIJSPath":"https://web.archive.org/web/20230306075114/https://ajax.googleapis.com/ajax/libs/jqueryui/1.12.0/jquery-ui.min.js"},"monitoring":{"clientTimingsAbsoluteTimeout":30000,"clientTimingsDebounceTimeout":1000},"mentions":{"maxNumUsersInDropdown":50},"markdown":{"enableTables":true},"legal":{"oneTrustConfigId":"cb0f3c87-b769-4e66-bbaa-377f9194216d"},"flags":{"allowRetractingCommentFlags":true,"allowRetractingFlags":true},"elections":{"opaVoteResultsBaseUrl":"https://web.archive.org/web/20230306075114/https://www.opavote.com/results/"},"comments":{},"accounts":{"currentPasswordRequiredForChangingStackIdPassword":true}}}</script> <script>StackExchange.init();</script> <script> StackExchange.using.setCacheBreakers({"Js/adops.en.js":"6da43f5e0a84","Js/ask.en.js":"","Js/begin-edit-event.en.js":"20edbaccceae","Js/copy-transpiled.en.js":"e7855bee94f2","Js/events.en.js":"","Js/explore-qlist.en.js":"2b1f34938b8b","Js/full-anon.en.js":"6a00afec4f5e","Js/full.en.js":"c8909108cd52","Js/highlightjs-loader.en.js":"a284064706b3","Js/inline-tag-editing.en.js":"629d801833ec","Js/keyboard-shortcuts.en.js":"4cb9eae35625","Js/markdown-it-loader.en.js":"5818ef89ff9d","Js/mentions-transpiled.en.js":"78637d7eac99","Js/moderator.en.js":"d88c424a0972","Js/postCollections-transpiled.en.js":"fac92aa0ceeb","Js/post-validation.en.js":"6da722d510e3","Js/question-editor.en.js":"","Js/review-v2-transpiled.en.js":"ae1e01b7e8cb","Js/revisions.en.js":"a86490719687","Js/stacks-editor.en.js":"4eaa25f823da","Js/tageditor.en.js":"e5db6df2411b","Js/tageditornew.en.js":"c4fc98223de1","Js/tagsuggestions.en.js":"0d912930b14c","Js/unlimited-transpiled.en.js":"7ed67670b600","Js/wmd.en.js":"67025ccb344d"}); StackExchange.using("gps", function() { StackExchange.gps.init(false); }); </script> <noscript id="noscript-css"><style>body,.s-topbar{margin-top:1.9em}</style></noscript> </head> <body class="question-page unified-theme"> <div id="notify-container"></div> <div id="custom-header"></div> <header class="s-topbar ps-fixed t0 l0 js-top-bar"> <div class="s-topbar--container"> <a href="#" class="s-topbar--menu-btn js-left-sidebar-toggle" role="menuitem" aria-haspopup="true" aria-controls="left-sidebar" aria-expanded="false"><span></span></a> <div class="topbar-dialog leftnav-dialog js-leftnav-dialog dno"> <div class="left-sidebar js-unpinned-left-sidebar" data-can-be="left-sidebar" data-is-here-when="sm"></div> </div> <a href="#" class="s-topbar--logo network-logo js-gps-track js-network-logo" data-gps-track="stack_exchange_popup.show" role="menuitem" aria-haspopup="true" aria-controls="topbar-network-logo-dialog" aria-expanded="false"> <svg aria-hidden="true" class="native mtn1 svg-icon iconLogoSEAlternativeSm" width="107" height="15" viewbox="0 0 107 15"><path d="m48.41 11.93-1.96-3.2-1.04 1.16v2.04h-1.42V2.18h1.42v6.01L48.14 5h1.72l-2.44 2.7 2.74 4.22h-1.75Zm-7.06.08c-1.59 0-3.14-.96-3.14-3.56s1.55-3.54 3.14-3.54c.97 0 1.65.27 2.31.97l-.97.93c-.44-.48-.79-.66-1.34-.66s-1 .22-1.3.62c-.31.38-.42.87-.42 1.68 0 .81.1 1.32.41 1.7.3.4.76.62 1.3.62.56 0 .9-.18 1.35-.66l.97.92c-.66.7-1.34.98-2.31.98Zm-5.66-3.15h-1.65c-.83 0-1.26.37-1.26 1s.4.99 1.3.99c.53 0 .93-.04 1.3-.4.22-.2.31-.53.31-1.03v-.56Zm.03 3.07v-.63c-.51.5-1 .71-1.87.71-.87 0-1.46-.2-1.89-.63a2.1 2.1 0 0 1-.55-1.49c0-1.16.82-2 2.42-2h1.86v-.5c0-.87-.44-1.3-1.54-1.3-.77 0-1.15.18-1.54.68l-.92-.86c.66-.77 1.35-1 2.52-1 1.93 0 2.9.8 2.9 2.38v4.64h-1.39Zm-5.9 0c-1.32 0-1.93-.93-1.93-1.93V6.18h-.8V5.1h.8V3h1.41v2.1h1.36v1.07H29.3v3.75c0 .5.25.81.78.81h.58v1.2h-.85Zm-6.33.08c-1.48 0-2.55-.34-3.49-1.28l1-.98c.72.72 1.51.94 2.52.94 1.3 0 2.04-.55 2.04-1.5 0-.42-.13-.78-.39-1.01-.25-.23-.5-.33-1.08-.41l-1.16-.17a3.4 3.4 0 0 1-1.88-.78 2.41 2.41 0 0 1-.72-1.86c0-1.7 1.25-2.86 3.3-2.86 1.3 0 2.22.33 3.07 1.1l-.96.94a2.92 2.92 0 0 0-2.15-.75c-1.16 0-1.8.65-1.8 1.52 0 .35.1.67.37.9.25.22.65.38 1.11.45l1.13.17c.91.13 1.42.35 1.84.72.54.47.8 1.17.8 2 0 1.8-1.48 2.86-3.55 2.86Z" fill="#FEFEFE"/><path d="M104.16 7.09c-.2-.42-.6-.74-1.2-.74s-.99.32-1.18.74c-.1.25-.15.44-.16.75h2.7a2 2 0 0 0-.16-.75Zm-2.54 1.96c0 .9.56 1.57 1.55 1.57.78 0 1.16-.21 1.61-.66l1.08 1.04a3.4 3.4 0 0 1-2.7 1.11c-1.68 0-3.29-.76-3.29-3.62 0-2.3 1.26-3.6 3.1-3.6 1.97 0 3.1 1.44 3.1 3.37v.79h-4.45Zm-5.48-2.57C95.1 6.48 95 7.37 95 8.3c0 .94.1 1.85 1.15 1.85 1.05 0 1.18-.91 1.18-1.85 0-.93-.13-1.82-1.18-1.82Zm-.17 8.22c-1.1 0-1.84-.21-2.58-.92l1.1-1.11c.4.38.8.54 1.4.54 1.06 0 1.43-.74 1.43-1.46v-.72c-.47.51-1 .7-1.7.7-.69 0-1.29-.23-1.68-.62-.67-.66-.73-1.57-.73-2.8 0-1.24.06-2.13.73-2.8.4-.39 1-.62 1.7-.62.75 0 1.24.2 1.73.75v-.67h1.72v6.8c0 1.7-1.21 2.93-3.12 2.93Zm-5.76-2.67V7.76c0-.96-.61-1.28-1.17-1.28-.56 0-1.18.32-1.18 1.28v4.27h-1.78V4.97h1.73v.65a2.44 2.44 0 0 1 1.78-.73c.7 0 1.28.23 1.67.62.58.57.73 1.24.73 2v4.52H90.2Zm-7.1-2.98h-1.4c-.64 0-1 .3-1 .8 0 .49.33.81 1.02.81.5 0 .8-.04 1.12-.34.2-.17.26-.46.26-.89v-.38Zm.04 2.98v-.6c-.48.47-.93.67-1.74.67-.8 0-1.4-.2-1.82-.62-.38-.4-.58-.97-.58-1.59 0-1.12.77-2.05 2.42-2.05h1.68V7.5c0-.77-.38-1.11-1.32-1.11-.68 0-1 .16-1.37.58l-1.13-1.1c.7-.75 1.38-.97 2.57-.97 1.99 0 3.02.84 3.02 2.5v4.64h-1.73Zm-6.93 0v-4.3c0-.94-.6-1.25-1.15-1.25-.56 0-1.15.32-1.15 1.24v4.31h-1.77V2.38h1.77v3.24a2.35 2.35 0 0 1 1.7-.73c1.56 0 2.38 1.08 2.38 2.57v4.57h-1.78Zm-6.96.08c-1.42 0-3.18-.76-3.18-3.62 0-2.85 1.76-3.6 3.18-3.6.98 0 1.72.3 2.34.95l-1.2 1.2c-.36-.4-.68-.56-1.14-.56-.42 0-.75.14-1.01.46-.27.33-.4.8-.4 1.55s.13 1.24.4 1.58c.26.3.59.46 1 .46.47 0 .79-.16 1.15-.56l1.2 1.18c-.62.65-1.36.96-2.34.96Zm-5.53-.08-1.3-2.11-1.3 2.11H59l2.45-3.6-2.35-3.46h2.12L62.42 7l1.21-2.02h2.13L63.4 8.43l2.46 3.6h-2.13Zm-11.75 0V2.06h6.6V3.8h-4.65v2.33h3.96v1.74h-3.96v2.42h4.65v1.74h-6.6Z" fill="#2F96E8"/><path d="M0 3c0-1.1.9-2 2-2h8a2 2 0 0 1 2 2H0Z" fill="#8FD8F7"/><path d="M12 10H0c0 1.1.9 2 2 2h5v3l3-3a2 2 0 0 0 2-2Z" fill="#155397"/><path fill="#46A2D9" d="M0 4h12v2H0z"/><path fill="#2D6DB5" d="M0 7h12v2H0z"/></svg> </a> <div class="topbar-dialog network-logo-dialog js-network-logo-dialog dno" id="topbar-network-logo-dialog" role="dialog" aria-labelledby="topbar-network-logo-dialog-title" aria-describedby="topbar-network-logo-dialog-body"> <div class="dialog-content"> <h4 class="bold" id="topbar-network-logo-dialog-title">Stack Exchange Network</h4> <p id="topbar-network-logo-dialog-body"> Stack Exchange network consists of 181 Q&amp;A communities including <a href="https://web.archive.org/web/20230306075114/https://stackoverflow.com/">Stack Overflow</a>, the largest, most trusted online community for developers to learn, share their knowledge, and build their careers. </p> <a class="s-btn s-btn__filled" href="https://web.archive.org/web/20230306075114/https://stackexchange.com/" data-gps-track="stack_exchange_popup.click">Visit Stack Exchange</a> <button class="icon-close js-close-button s-btn s-btn__unset" aria-label="Close"><svg aria-hidden="true" class="svg-icon iconClear" width="18" height="18" viewbox="0 0 18 18"><path d="M15 4.41 13.59 3 9 7.59 4.41 3 3 4.41 7.59 9 3 13.59 4.41 15 9 10.41 13.59 15 15 13.59 10.41 9 15 4.41Z"/></svg></button> </div> </div> <form id="search" role="search" action="/web/20230306075114/https://diy.stackexchange.com/search" class="s-topbar--searchbar js-searchbar " autocomplete="off"> <div class="s-topbar--searchbar--input-group"> <input name="q" type="text" role="combobox" placeholder="Search on Home Improvement…" value="" autocomplete="off" maxlength="240" class="s-input s-input__search js-search-field " aria-label="Search" aria-controls="top-search" data-controller="s-popover" data-action="focus->s-popover#show" data-s-popover-placement="bottom-start"/> <svg aria-hidden="true" class="s-input-icon s-input-icon__search svg-icon iconSearch" width="18" height="18" viewbox="0 0 18 18"><path d="m18 16.5-5.14-5.18h-.35a7 7 0 1 0-1.19 1.19v.35L16.5 18l1.5-1.5ZM12 7A5 5 0 1 1 2 7a5 5 0 0 1 10 0Z"/></svg> <div class="s-popover p0 wmx100 wmn4 sm:wmn-initial js-top-search-popover" id="top-search" role="menu"> <div class="s-popover--arrow"></div> <div class="js-spinner p24 d-flex ai-center jc-center d-none"> <div class="s-spinner s-spinner__sm fc-orange-400"> <div class="v-visible-sr">Loading&#x2026;</div> </div> </div> <span class="v-visible-sr js-screen-reader-info"></span> <div class="js-ac-results overflow-y-auto hmx3 d-none"></div> <div class="js-search-hints" aria-describedby="Tips for searching"></div> </div> </div> </form> <nav class="h100 ml-auto overflow-x-auto pr12"> <ol class="s-topbar--content" role="menubar"> <li role="none"> <a href="/web/20230306075114/https://diy.stackexchange.com/help" class="s-topbar--item js-help-button" role="menuitem" title="Help Center and other resources" aria-haspopup="true" aria-controls="topbar-help-dialog" data-ga="[&quot;top navigation&quot;,&quot;help menu click&quot;,null,null,null]"><svg aria-hidden="true" class="svg-icon iconHelp" width="18" height="18" viewbox="0 0 18 18"><path d="M9 1C4.64 1 1 4.64 1 9c0 4.36 3.64 8 8 8 4.36 0 8-3.64 8-8 0-4.36-3.64-8-8-8Zm.81 12.13c-.02.71-.55 1.15-1.24 1.13-.66-.02-1.17-.49-1.15-1.2.02-.72.56-1.18 1.22-1.16.7.03 1.2.51 1.17 1.23ZM11.77 8c-.59.66-1.78 1.09-2.05 1.97a4 4 0 0 0-.09.75c0 .05-.03.16-.18.16H7.88c-.16 0-.18-.1-.18-.15.06-1.35.66-2.2 1.83-2.88.39-.29.7-.75.7-1.24.01-1.24-1.64-1.82-2.35-.72-.21.33-.18.73-.18 1.1H5.75c0-1.97 1.03-3.26 3.03-3.26 1.75 0 3.47.87 3.47 2.83 0 .57-.2 1.05-.48 1.44Z"/></svg></a> </li> <div class="topbar-dialog help-dialog js-help-dialog dno" id="topbar-help-dialog" role="menu"> <div class="modal-content"> <ul> <li> <a href="/web/20230306075114/https://diy.stackexchange.com/tour" class="js-gps-track" data-gps-track="help_popup.click({ item_type:1 })" data-ga="[&quot;top navigation&quot;,&quot;tour submenu click&quot;,null,null,null]"> Tour <span class="item-summary"> Start here for a quick overview of the site </span> </a> </li> <li> <a href="/web/20230306075114/https://diy.stackexchange.com/help" class="js-gps-track" data-gps-track="help_popup.click({ item_type:4 })" data-ga="[&quot;top navigation&quot;,&quot;help center&quot;,null,null,null]"> Help Center <span class="item-summary"> Detailed answers to any questions you might have </span> </a> </li> <li> <a href="https://web.archive.org/web/20230306075114/https://diy.meta.stackexchange.com/" class="js-gps-track" data-gps-track="help_popup.click({ item_type:2 })" data-ga="[&quot;top navigation&quot;,&quot;meta submenu click&quot;,null,null,null]"> Meta <span class="item-summary"> Discuss the workings and policies of this site </span> </a> </li> <li> <a href="https://web.archive.org/web/20230306075114/https://stackoverflow.co/" class="js-gps-track" data-gps-track="help_popup.click({ item_type:6 })" data-ga="[&quot;top navigation&quot;,&quot;about us submenu click&quot;,null,null,null]"> About Us <span class="item-summary"> Learn more about Stack Overflow the company, and our products. </span> </a> </li> </ul> </div> </div> <li role="none"> <a href="https://web.archive.org/web/20230306075114/https://stackexchange.com/" class="s-topbar--item js-site-switcher-button js-gps-track" data-gps-track="site_switcher.show" aria-label="Site switcher" role="menuitem" title="A list of all 181 Stack Exchange sites" aria-haspopup="true" aria-expanded="false" data-ga="[&quot;top navigation&quot;,&quot;stack exchange click&quot;,null,null,null]"> <svg aria-hidden="true" class="svg-icon iconStackExchange" width="18" height="18" viewbox="0 0 18 18"><path d="M15 1H3a2 2 0 0 0-2 2v2h16V3a2 2 0 0 0-2-2ZM1 13c0 1.1.9 2 2 2h8v3l3-3h1a2 2 0 0 0 2-2v-2H1v2Zm16-7H1v4h16V6Z"/></svg> </a> </li> <li class="js-topbar-dialog-corral" role="presentation"> <div class="topbar-dialog siteSwitcher-dialog dno" role="menu"> <div class="header fw-wrap"> <h3 class="flex--item"> <a href="https://web.archive.org/web/20230306075114/https://diy.stackexchange.com/">current community</a> </h3> <div class="flex--item fl1"> <div class="ai-center d-flex jc-end"> <button class="js-close-button s-btn s-btn__muted p0 ml8 d-none sm:d-block" type="button" aria-label="Close"> <svg aria-hidden="true" class="svg-icon iconClear" width="18" height="18" viewbox="0 0 18 18"><path d="M15 4.41 13.59 3 9 7.59 4.41 3 3 4.41 7.59 9 3 13.59 4.41 15 9 10.41 13.59 15 15 13.59 10.41 9 15 4.41Z"/></svg> </button> </div> </div> </div> <div class="modal-content bg-powder-050 current-site-container"> <ul class="current-site "> <li class="d-flex"> <div class="fl1"> <a href="https://web.archive.org/web/20230306075114/https://diy.stackexchange.com/" class="current-site-link site-link js-gps-track d-flex gs8 gsx" data-id="73" data-gps-track="site_switcher.click({ item_type:3 })"> <div class="favicon favicon-diy site-icon flex--item" title="Home Improvement"></div> <span class="flex--item fl1"> Home Improvement </span> </a> </div> <div class="related-links"> <a href="https://web.archive.org/web/20230306075114/https://diy.stackexchange.com/help" class="js-gps-track" data-gps-track="site_switcher.click({ item_type:14 })">help</a> <a href="https://web.archive.org/web/20230306075114/https://chat.stackexchange.com/?tab=site&amp;host=diy.stackexchange.com" class="js-gps-track" data-gps-track="site_switcher.click({ item_type:6 })">chat</a> </div> </li> <li class="related-site d-flex"> <div class="L-shaped-icon-container"> <span class="L-shaped-icon"></span> </div> <a href="https://web.archive.org/web/20230306075114/https://diy.meta.stackexchange.com/" class=" site-link js-gps-track d-flex gs8 gsx" data-id="74" data-gps-track="site.switch({ target_site:74, item_type:3 }),site_switcher.click({ item_type:4 })"> <div class="favicon favicon-diymeta site-icon flex--item" title="Home Improvement Meta"></div> <span class="flex--item fl1"> Home Improvement Meta </span> </a> </li> </ul> </div> <div class="header" id="your-communities-header"> <h3> your communities </h3> </div> <div class="modal-content" id="your-communities-section"> <div class="call-to-login"> <a href="https://web.archive.org/web/20230306075114/https://diy.stackexchange.com/users/signup?ssrc=site_switcher&amp;returnurl=https%3a%2f%2fdiy.stackexchange.com%2fquestions%2f268276%2fwhat-kind-of-wall-is-this-and-how-can-i-repair-it" class="login-link js-gps-track" data-gps-track="site_switcher.click({ item_type:10 })">Sign up</a> or <a href="https://web.archive.org/web/20230306075114/https://diy.stackexchange.com/users/login?ssrc=site_switcher&amp;returnurl=https%3a%2f%2fdiy.stackexchange.com%2fquestions%2f268276%2fwhat-kind-of-wall-is-this-and-how-can-i-repair-it" class="login-link js-gps-track" data-gps-track="site_switcher.click({ item_type:11 })">log in</a> to customize your list. </div> </div> <div class="header"> <h3><a href="https://web.archive.org/web/20230306075114/https://stackexchange.com/sites">more stack exchange communities</a> </h3> <a href="https://web.archive.org/web/20230306075114/https://stackoverflow.blog/" class="float-right">company blog</a> </div> <div class="modal-content"> <div class="child-content"></div> </div> </div> </li> <li role="none"><button class="s-topbar--item s-btn s-btn__icon s-btn__muted d-none sm:d-inline-flex js-searchbar-trigger" role="menuitem" aria-label="Search" aria-haspopup="true" aria-controls="search" title="Click to show search"><svg aria-hidden="true" class="svg-icon iconSearch" width="18" height="18" viewbox="0 0 18 18"><path d="m18 16.5-5.14-5.18h-.35a7 7 0 1 0-1.19 1.19v.35L16.5 18l1.5-1.5ZM12 7A5 5 0 1 1 2 7a5 5 0 0 1 10 0Z"/></svg></button></li> <li role="none"> <a href="https://web.archive.org/web/20230306075114/https://diy.stackexchange.com/users/login?ssrc=head&amp;returnurl=https%3a%2f%2fdiy.stackexchange.com%2fquestions%2f268276%2fwhat-kind-of-wall-is-this-and-how-can-i-repair-it" class="s-topbar--item s-topbar--item__unset s-btn s-btn__filled ws-nowrap js-gps-track" role="menuitem" rel="nofollow" data-gps-track="login.click" data-ga="[&quot;top navigation&quot;,&quot;login button click&quot;,null,null,null]">Log in</a> </li> <li role="none"><a href="https://web.archive.org/web/20230306075114/https://diy.stackexchange.com/users/signup?ssrc=head&amp;returnurl=https%3a%2f%2fdiy.stackexchange.com%2fquestions%2f268276%2fwhat-kind-of-wall-is-this-and-how-can-i-repair-it" class="s-topbar--item s-topbar--item__unset ml4 s-btn s-btn__primary ws-nowrap" role="menuitem" rel="nofollow" data-ga="[&quot;sign up&quot;,&quot;Sign Up Navigation&quot;,&quot;Header&quot;,null,null]">Sign up</a></li> </ol> </nav> </div> </header> <script> StackExchange.ready(function () { StackExchange.topbar.init(); }); StackExchange.scrollPadding.setPaddingTop(50, 10); </script> <div class="sm:d-none py24 bg-black-750 fc-black-200 ps-relative js-dismissable-hero"> <div class="px12 d-flex ai-center jc-center mx-auto wmx12"> <div class="flex--item wmx3 fs-body2 mr64 md:mr32"> <p>Home Improvement Stack Exchange is a question and answer site for contractors and serious DIYers. It only takes a minute to sign up.</p> <a href="/web/20230306075114/https://diy.stackexchange.com/users/signup?ssrc=hero&amp;returnurl=https%3a%2f%2fdiy.stackexchange.com%2fquestions%2f268276%2fwhat-kind-of-wall-is-this-and-how-can-i-repair-it" class="s-btn s-btn__primary">Sign up to join this community</a> </div> <div class="d-flex fd-column ai-center wmn3 hero-background"> <div class="d-flex ai-center mb24"> <div class="flex--item mr16"> <img width="31" src="https://web.archive.org/web/20230306075114im_/https://cdn.sstatic.net/Img/hero/anonymousHeroQuestions.svg?v=748bfb046b78" alt=""> </div> <div class="flex--item"> Anybody can ask a question </div> </div> <div class="d-flex ai-center mb24"> <div class="flex--item mr16"> <img width="35" src="https://web.archive.org/web/20230306075114im_/https://cdn.sstatic.net/Img/hero/anonymousHeroAnswers.svg?v=d5348b00eddc" alt=""> </div> <div class="flex--item"> Anybody can answer </div> </div> <div class="d-flex ai-center"> <div class="flex--item mr16"> <img width="24" src="https://web.archive.org/web/20230306075114im_/https://cdn.sstatic.net/Img/hero/anonymousHeroUpvote.svg?v=af2bb70d5d1b" alt=""> </div> <div class="flex--item wmx2"> The best answers are voted up and rise to the top </div> </div> </div> <div class="flex--item as-start md:ps-absolute t8 r8"> <button class="s-btn s-btn__muted p8 js-dismiss"> <svg aria-hidden="true" class="svg-icon iconClear" width="18" height="18" viewbox="0 0 18 18"><path d="M15 4.41 13.59 3 9 7.59 4.41 3 3 4.41 7.59 9 3 13.59 4.41 15 9 10.41 13.59 15 15 13.59 10.41 9 15 4.41Z"/></svg> </button> </div> </div> </div> <script> StackExchange.ready(function () { StackExchange.Hero.init("nso", "a"); var location = 0; if ($("body").hasClass("questions-page")) { location = 1; } else if ($("body").hasClass("question-page")) { location = 1; } else if ($("body").hasClass("faq-page")) { location = 5; } else if ($("body").hasClass("home-page")) { location = 3; } $('.js-cta-button').click(function () { StackExchange.using("gps", function () { StackExchange.gps.track("hero.action", { hero_action_type: 'cta', location: location }, true); }); }); // TODO: we should review the class names and whatnot in use here. Older heroes use id selectors, the newer // sticky question hero on SO has a .js-dismiss class instead, but it's apparently not used anywhere... // It's not great. Ideally we'd have a set of classes in the partials above that would correspond to // the behaviours we want here in a more clear way. // sticky question-page hero at the bottom of the page on SO $('.js-dismiss').on('click', function () { StackExchange.using("gps", function () { StackExchange.gps.track("hero.action", { hero_action_type: "close", location: location }, true); }); StackExchange.Hero.dismiss(); $(".js-dismissable-hero").fadeOut("fast"); }); }); </script> <header class="site-header"> <div class="site-header--container"> <a class="site-header--link d-flex ai-center fs-headline1 fw-bold" href="https://web.archive.org/web/20230306075114/https://diy.stackexchange.com/"> <img class="h-auto wmx100" src="https://web.archive.org/web/20230306075114im_/https://cdn.sstatic.net/Sites/diy/Img/logo.svg?v=adde20d51da3" alt="Home Improvement"> </a> </div> </header> <div class="container"> <div id="left-sidebar" data-is-here-when="md lg" class="left-sidebar js-pinned-left-sidebar ps-relative"> <div class="left-sidebar--sticky-container js-sticky-leftnav"> <nav role="navigation"> <ol class="nav-links"> <li class="ps-relative" aria-current="false"> <a href="/web/20230306075114/https://diy.stackexchange.com/" class="pl8 js-gps-track nav-links--link" data-gps-track="top_nav.click({is_current: false, location:2, destination:8})" aria-controls="" data-controller="" data-s-popover-placement="right" aria-current="false" data-s-popover-auto-show="true" data-s-popover-hide-on-outside-click="never"> <div class="d-flex ai-center"> <div class="flex--item truncate"> Home </div> </div> </a> </li> <li> <ol class="nav-links"> <li class="fs-fine tt-uppercase ml8 mt16 mb4 fc-light">Public</li> <li class="ps-relative youarehere" aria-current="true"> <a id="nav-questions" href="/web/20230306075114/https://diy.stackexchange.com/questions" class="pl8 js-gps-track nav-links--link -link__with-icon" data-gps-track="top_nav.click({is_current: true, location:2, destination:1})" aria-controls="" data-controller="" data-s-popover-placement="right" aria-current="false" data-s-popover-auto-show="true" data-s-popover-hide-on-outside-click="never"> <svg aria-hidden="true" class="svg-icon iconGlobe" width="18" height="18" viewbox="0 0 18 18"><path d="M9 1C4.64 1 1 4.64 1 9c0 4.36 3.64 8 8 8 4.36 0 8-3.64 8-8 0-4.36-3.64-8-8-8ZM8 15.32a6.46 6.46 0 0 1-4.3-2.74 6.46 6.46 0 0 1-.93-5.01L7 11.68v.8c0 .88.12 1.32 1 1.32v1.52Zm5.72-2c-.2-.66-1-1.32-1.72-1.32h-1v-2c0-.44-.56-1-1-1H6V7h1c.44 0 1-.56 1-1V5h2c.88 0 1.4-.72 1.4-1.6v-.33a6.45 6.45 0 0 1 3.83 4.51 6.45 6.45 0 0 1-1.51 5.73v.01Z"/></svg> <span class="-link--channel-name">Questions</span> </a> </li> <li class="ps-relative" aria-current="false"> <a id="nav-tags" href="/web/20230306075114/https://diy.stackexchange.com/tags" class=" js-gps-track nav-links--link" data-gps-track="top_nav.click({is_current: false, location:2, destination:2})" aria-controls="" data-controller="" data-s-popover-placement="right" aria-current="false" data-s-popover-auto-show="true" data-s-popover-hide-on-outside-click="never"> <div class="d-flex ai-center"> <div class="flex--item truncate"> Tags </div> </div> </a> </li> <li class="ps-relative" aria-current="false"> <a id="nav-users" href="/web/20230306075114/https://diy.stackexchange.com/users" class=" js-gps-track nav-links--link" data-gps-track="top_nav.click({is_current: false, location:2, destination:3})" aria-controls="" data-controller="" data-s-popover-placement="right" aria-current="false" data-s-popover-auto-show="true" data-s-popover-hide-on-outside-click="never"> <div class="d-flex ai-center"> <div class="flex--item truncate"> Users </div> </div> </a> </li> <li class="ps-relative" aria-current="false"> <a id="nav-unanswered" href="/web/20230306075114/https://diy.stackexchange.com/unanswered" class=" js-gps-track nav-links--link" data-gps-track="top_nav.click({is_current: false, location:2, destination:5})" aria-controls="" data-controller="" data-s-popover-placement="right" aria-current="false" data-s-popover-auto-show="true" data-s-popover-hide-on-outside-click="never"> <div class="d-flex ai-center"> <div class="flex--item truncate"> Unanswered </div> </div> </a> </li> </ol> </li> <li> <ol class="nav-links"> <li class="js-freemium-cta ps-relative"> <div class="fs-fine tt-uppercase ml8 mt16 mb8 fc-light">Teams</div> <div class="bt bl bb bc-black-075 p12 pb6 fc-black-600 blr-sm overflow-hidden"> <strong class="fc-black-750 mb6">Stack Overflow for Teams</strong> – Start collaborating and sharing organizational knowledge. <img class="wmx100 mx-auto my8 h-auto d-block" width="139" height="114" src="https://web.archive.org/web/20230306075114im_/https://cdn.sstatic.net/Img/teams/teams-illo-free-sidebar-promo.svg?v=47faa659a05e" alt=""> <a href="https://web.archive.org/web/20230306075114/https://try.stackoverflow.co/why-teams/?utm_source=so-owned&amp;utm_medium=side-bar&amp;utm_campaign=campaign-38&amp;utm_content=cta" class="w100 s-btn s-btn__primary s-btn__xs bg-orange-400 js-gps-track" data-gps-track="teams.create.left-sidenav.click({ Action: 6 })" data-ga="[&quot;teams left navigation - anonymous&quot;,&quot;left nav free cta&quot;,&quot;stackoverflow.com/teams/create/free&quot;,null,null]">Create a free Team</a> <a href="https://web.archive.org/web/20230306075114/https://stackoverflow.co/teams" class="w100 s-btn s-btn__muted s-btn__xs js-gps-track" data-gps-track="teams.create.left-sidenav.click({ Action: 5 })" data-ga="[&quot;teams left navigation - anonymous&quot;,&quot;left nav free cta&quot;,&quot;stackoverflow.com/teams&quot;,null,null]">Why Teams?</a> </div> </li> <li class="d-flex ai-center jc-space-between ml8 mt24 mb4 js-create-team-cta d-none"> <div class="flex--item tt-uppercase fs-fine fc-light">Teams</div> <div class="flex--item"> <a href="javascript:void(0)" class="s-link p12 fc-black-500 h:fc-black-800 js-gps-track" role="button" aria-controls="popover-teams-create-cta" data-controller="s-popover" data-action="s-popover#toggle" data-s-popover-placement="bottom-start" data-s-popover-toggle-class="is-selected" data-gps-track="teams.create.left-sidenav.click({ Action: ShowInfo })" data-ga="[&quot;teams left navigation - anonymous&quot;,&quot;left nav show teams info&quot;,null,null,null]"> <svg aria-hidden="true" class="svg-icon iconInfoSm" width="14" height="14" viewbox="0 0 14 14"><path d="M7 1a6 6 0 1 1 0 12A6 6 0 0 1 7 1Zm1 10V6H6v5h2Zm0-6V3H6v2h2Z"/></svg> </a> </div> </li> <li class="ps-relative js-create-team-cta d-none"> <a href="https://web.archive.org/web/20230306075114/https://stackoverflowteams.com/teams/create/free?utm_source=so-owned&amp;utm_medium=side-bar&amp;utm_campaign=campaign-38&amp;utm_content=cta" class="pl8 js-gps-track nav-links--link" title="Stack Overflow for Teams is a private, secure spot for your organization's questions and answers." data-gps-track="teams.create.left-sidenav.click({ Action: FreemiumTeamsCreateClick })" data-ga="[&quot;teams left navigation - anonymous&quot;,&quot;left nav team click&quot;,&quot;stackoverflow.com/teams/create/free&quot;,null,null]"> <div class="d-flex ai-center"> <div class="flex--item s-avatar va-middle bg-orange-400"> <div class="s-avatar--letter mtn1"> <svg aria-hidden="true" class="svg-icon iconBriefcaseSm" width="14" height="14" viewbox="0 0 14 14"><path d="M4 3a1 1 0 0 1 1-1h4a1 1 0 0 1 1 1v1h.5c.83 0 1.5.67 1.5 1.5v5c0 .83-.67 1.5-1.5 1.5h-7A1.5 1.5 0 0 1 2 10.5v-5C2 4.67 2.67 4 3.5 4H4V3Zm5 1V3H5v1h4Z"/></svg> </div> <svg aria-hidden="true" class="native s-avatar--badge svg-icon iconShieldXSm" width="9" height="10" viewbox="0 0 9 10"><path fill="var(--white)" d="M0 1.84 4.5 0 9 1.84v3.17C9 7.53 6.3 10 4.5 10 2.7 10 0 7.53 0 5.01V1.84Z"/><path fill="var(--black-500)" d="M1 2.5 4.5 1 8 2.5v2.51C8 7.34 5.34 9 4.5 9 3.65 9 1 7.34 1 5.01V2.5Zm2.98 3.02L3.2 7h2.6l-.78-1.48a.4.4 0 0 1 .15-.38c.34-.24.73-.7.73-1.14 0-.71-.5-1.23-1.41-1.23-.92 0-1.39.52-1.39 1.23 0 .44.4.9.73 1.14.12.08.18.23.15.38Z"/></svg> </div> <div class="flex--item pl6"> Create free Team </div> </div> </a> </li> </ol> </li> </ol> </nav> </div> <div class="s-popover ws2" id="popover-teams-create-cta" role="menu" aria-hidden="true"> <div class="s-popover--arrow"></div> <div class="ps-relative overflow-hidden"> <p class="mb2"><strong>Teams</strong></p> <p class="mb12 fs-caption fc-black-400">Q&amp;A for work</p> <p class="mb12 fs-caption fc-medium">Connect and share knowledge within a single location that is structured and easy to search.</p> <a href="https://web.archive.org/web/20230306075114/https://stackoverflow.co/teams" class="js-gps-track s-btn s-btn__primary s-btn__xs" data-gps-track="teams.create.left-sidenav.click({ Action: CtaClick })" data-ga="[&quot;teams left navigation - anonymous&quot;,&quot;left nav cta&quot;,&quot;stackoverflow.com/teams&quot;,null,null]"> Learn more about Teams </a> </div> <div class="ps-absolute t8 r8"> <svg aria-hidden="true" class="fc-orange-500 svg-spot spotPeople" width="48" height="48" viewbox="0 0 48 48"><path d="M13.5 28a4.5 4.5 0 1 0 0-9 4.5 4.5 0 0 0 0 9ZM7 30a1 1 0 0 1 1-1h11a1 1 0 0 1 1 1v5h11v-5a1 1 0 0 1 1-1h12a1 1 0 0 1 1 1v10a2 2 0 0 1-2 2H33v5a1 1 0 0 1-1 1H20a1 1 0 0 1-1-1v-5H8a1 1 0 0 1-1-1V30Zm25-6.5a4.5 4.5 0 1 0 9 0 4.5 4.5 0 0 0-9 0ZM24.5 34a4.5 4.5 0 1 0 0-9 4.5 4.5 0 0 0 0 9Z" opacity=".2"/><path d="M16.4 26.08A6 6 0 1 0 7.53 26C5.64 26.06 4 27.52 4 29.45V40a1 1 0 0 0 1 1h9a1 1 0 1 0 0-2h-4v-7a1 1 0 1 0-2 0v7H6v-9.55c0-.73.67-1.45 1.64-1.45H16a1 1 0 0 0 .4-1.92ZM12 18a4 4 0 1 1 0 8 4 4 0 0 1 0-8Zm16.47 14a6 6 0 1 0-8.94 0A3.6 3.6 0 0 0 16 35.5V46a1 1 0 0 0 1 1h14a1 1 0 0 0 1-1V35.5c0-1.94-1.64-3.42-3.53-3.5ZM20 28a4 4 0 1 1 8 0 4 4 0 0 1-8 0Zm-.3 6h8.6c1 0 1.7.75 1.7 1.5V45h-2v-7a1 1 0 1 0-2 0v7h-4v-7a1 1 0 1 0-2 0v7h-2v-9.5c0-.75.7-1.5 1.7-1.5ZM42 22c0 1.54-.58 2.94-1.53 4A3.5 3.5 0 0 1 44 29.45V40a1 1 0 0 1-1 1h-9a1 1 0 1 1 0-2h4v-7a1 1 0 1 1 2 0v7h2v-9.55A1.5 1.5 0 0 0 40.48 28H32a1 1 0 0 1-.4-1.92A6 6 0 1 1 42 22Zm-2 0a4 4 0 1 0-8 0 4 4 0 0 0 8 0Z"/><g opacity=".35"><path d="M17 10a1 1 0 011-1h12a1 1 0 110 2H18a1 1 0 01-1-1Zm1-5a1 1 0 100 2h12a1 1 0 100-2H18ZM14 1a1 1 0 00-1 1v12a1 1 0 001 1h5.09l4.2 4.2a1 1 0 001.46-.04l3.7-4.16H34a1 1 0 001-1V2a1 1 0 00-1-1H14Zm1 12V3h18v10h-5a1 1 0 00-.75.34l-3.3 3.7-3.74-3.75a1 1 0 00-.71-.29H15Z"/></g></svg> </div> </div> </div> <div id="content" class=""> <div itemprop="mainEntity" itemscope itemtype="https://schema.org/Question"> <link itemprop="image" href="https://cdn.sstatic.net/Sites/diy/Img/apple-touch-icon.png?v=26d138b3a1da"> <div class="inner-content clearfix"> <div id="question-header" class="d-flex sm:fd-column"> <h1 itemprop="name" class="fs-headline1 ow-break-word mb8 flex--item fl1"><a href="/web/20230306075114/https://diy.stackexchange.com/questions/268276/what-kind-of-wall-is-this-and-how-can-i-repair-it" class="question-hyperlink">What kind of wall is this and how can I repair it?</a></h1> <div class="ml12 aside-cta flex--item print:d-none sm:ml0 sm:mb12 sm:order-first sm:as-end"> <a href="/web/20230306075114/https://diy.stackexchange.com/questions/ask" class="ws-nowrap s-btn s-btn__primary"> Ask Question </a> </div> </div> <div class="d-flex fw-wrap pb8 mb16 bb bc-black-075"> <div class="flex--item ws-nowrap mr16 mb8" title="2023-03-05 19:15:18Z"> <span class="fc-light mr2">Asked</span> <time itemprop="dateCreated" datetime="2023-03-05T19:15:18">today</time> </div> <div class="flex--item ws-nowrap mr16 mb8"> <span class="fc-light mr2">Modified</span> <a href="?lastactivity" class="s-link s-link__inherit" title="2023-03-05 23:50:55Z">today</a> </div> <div class="flex--item ws-nowrap mb8" title="Viewed 175 times"> <span class="fc-light mr2">Viewed</span> 175 times </div> </div> <div id="mainbar" role="main" aria-label="question and answers"> <div class="question js-question" data-questionid="268276" data-position-on-page="0" data-score="3" id="question"> <style> </style> <div class="js-zone-container zone-container-main"> <div id="dfp-tlb" class="everyonelovesstackoverflow everyoneloves__top-leaderboard everyoneloves__leaderboard"></div> <div class="js-report-ad-button-container " style="width: 728px"></div> </div> <div class="post-layout "> <div class="votecell post-layout--left"> <div class="js-voting-container d-flex jc-center fd-column ai-stretch gs4 fc-black-200" data-post-id="268276"> <button class="js-vote-up-btn flex--item s-btn s-btn__unset c-pointer " data-controller="s-tooltip" data-s-tooltip-placement="right" title="This question shows research effort; it is useful and clear" aria-pressed="false" aria-label="Up vote" data-selected-classes="fc-theme-primary" data-unselected-classes=""> <svg aria-hidden="true" class="svg-icon iconArrowUpLg" width="36" height="36" viewbox="0 0 36 36"><path d="M2 25h32L18 9 2 25Z"/></svg> </button> <div class="js-vote-count flex--item d-flex fd-column ai-center fc-black-500 fs-title" itemprop="upvoteCount" data-value="3"> 3 </div> <button class="js-vote-down-btn flex--item s-btn s-btn__unset c-pointer " data-controller="s-tooltip" data-s-tooltip-placement="right" title="This question does not show any research effort; it is unclear or not useful" aria-pressed="false" aria-label="Down vote" data-selected-classes="fc-theme-primary" data-unselected-classes=""> <svg aria-hidden="true" class="svg-icon iconArrowDownLg" width="36" height="36" viewbox="0 0 36 36"><path d="M2 11h32L18 27 2 11Z"/></svg> </button> <button class="js-saves-btn s-btn s-btn__unset c-pointer py4" id="saves-btn-268276" data-controller="s-tooltip" data-s-tooltip-placement="right" data-s-popover-placement="" title="Save this question." aria-pressed="false" data-post-id="268276" data-post-type-id="1" data-user-privilege-for-post-click="0" aria-controls="" data-s-popover-auto-show="false"> <svg aria-hidden="true" class="fc-theme-primary-500 js-saves-btn-selected d-none svg-icon iconBookmark" width="18" height="18" viewbox="0 0 18 18"><path d="M3 17V3c0-1.1.9-2 2-2h8a2 2 0 0 1 2 2v14l-6-4-6 4Z"/></svg> <svg aria-hidden="true" class="js-saves-btn-unselected svg-icon iconBookmarkAlt" width="18" height="18" viewbox="0 0 18 18"><path d="m9 10.6 4 2.66V3H5v10.26l4-2.66ZM3 17V3c0-1.1.9-2 2-2h8a2 2 0 0 1 2 2v14l-6-4-6 4Z"/></svg> </button> <a class="js-post-issue flex--item s-btn s-btn__unset c-pointer py6 mx-auto" href="/web/20230306075114/https://diy.stackexchange.com/posts/268276/timeline" data-shortcut="T" data-ks-title="timeline" data-controller="s-tooltip" data-s-tooltip-placement="right" title="Show activity on this post." aria-label="Timeline"><svg aria-hidden="true" class="mln2 mr0 svg-icon iconHistory" width="19" height="18" viewbox="0 0 19 18"><path d="M3 9a8 8 0 1 1 3.73 6.77L8.2 14.3A6 6 0 1 0 5 9l3.01-.01-4 4-4-4h3L3 9Zm7-4h1.01L11 9.36l3.22 2.1-.6.93L10 10V5Z"/></svg></a> </div> </div> <div class="postcell post-layout--right"> <div class="s-prose js-post-body" itemprop="text"> <p><img src="https://web.archive.org/web/20230306075114im_/https://imgur.com/5jbaTKz.jpg" alt="1"/></p> <p><img src="https://web.archive.org/web/20230306075114im_/https://imgur.com/m9Q7276.jpg" alt="2"/></p> <p><img src="https://web.archive.org/web/20230306075114im_/https://imgur.com/jg5p60z.jpg" alt="3"/></p> <p>Album <a href="https://web.archive.org/web/20230306075114/https://imgur.com/a/tpbF8wL" rel="nofollow noreferrer">here</a>.</p> <p>In the master bedroom of my house, the front wall has some surface cracks that I'm trying to address before repainting. For some background info, this is an early 1900s rowhome. The interior of the house is drywall except for this particular wall in this room. There are no studs and knocking on the wall at various spots indicates to me that the wall is completely solid.</p> <p>I stuck my finger under one of the cracks and a piece just flaked right off -- it's definitely not drywall, and I don't think it's plaster and lath, but I'm pretty unfamiliar with that kind of wall so I could be mistaken. There seems to be 3 layers -- the paint, a very thin layer of something that has a rust colored backing to it, and then what I thought was concrete until I started poking at it. As shown in the pictures, there was a hole that someone drilled into the wall, so I stuck a screwdriver in there and started scraping around -- it basically disintegrated into a powder. It looks to be about as thick as a piece of drywall, but it's made of something I'm not familiar with. Behind <em>that</em>, I'm not really sure.</p> <p>Any ideas on the structure of this wall and what I can do to fix it? I've already done some drywall repairs with joint compound, but this is a bit out of my wheelhouse. I'm happy to take more pictures if they are needed.</p> </div> <div class="mt24 mb12"> <div class="post-taglist d-flex gs4 gsy fd-column"> <div class="d-flex ps-relative fw-wrap"> <ul class="ml0 list-ls-none js-post-tag-list-wrapper d-inline"><li class="d-inline mr4 js-post-tag-list-item"><a href="/web/20230306075114/https://diy.stackexchange.com/questions/tagged/repair" class="post-tag" title="show questions tagged 'repair'" aria-label="show questions tagged 'repair'" rel="tag" aria-labelledby="tag-repair-tooltip-container">repair</a></li><li class="d-inline mr4 js-post-tag-list-item"><a href="/web/20230306075114/https://diy.stackexchange.com/questions/tagged/walls" class="post-tag" title="show questions tagged 'walls'" aria-label="show questions tagged 'walls'" rel="tag" aria-labelledby="tag-walls-tooltip-container">walls</a></li></ul> </div> </div> </div> <div class="mb0 "> <div class="mt16 d-flex gs8 gsy fw-wrap jc-end ai-start pt4 mb16"> <div class="flex--item mr16 fl1 w96"> <div class="js-post-menu pt2" data-post-id="268276" data-post-type-id="1"> <div class="d-flex gs8 s-anchors s-anchors__muted fw-wrap"> <div class="flex--item"> <a href="/web/20230306075114/https://diy.stackexchange.com/q/268276" rel="nofollow" itemprop="url" class="js-share-link js-gps-track" title="Short permalink to this question" data-gps-track="post.click({ item: 2, priv: 0, post_type: 1 })" data-controller="se-share-sheet" data-se-share-sheet-title="Share a link to this question" data-se-share-sheet-subtitle="" data-se-share-sheet-post-type="question" data-se-share-sheet-social="facebook twitter " data-se-share-sheet-location="1" data-se-share-sheet-license-url="https%3a%2f%2fcreativecommons.org%2flicenses%2fby-sa%2f4.0%2f" data-se-share-sheet-license-name="CC BY-SA 4.0" data-s-popover-placement="bottom-start">Share</a> </div> <div class="flex--item"> <a href="/web/20230306075114/https://diy.stackexchange.com/posts/268276/edit" class="js-suggest-edit-post js-gps-track" data-gps-track="post.click({ item: 6, priv: 0, post_type: 1 })" title="">Improve this question</a> </div> <div class="flex--item"> <button type="button" id="btnFollowPost-268276" class="s-btn s-btn__link js-follow-post js-follow-question js-gps-track" data-gps-track="post.click({ item: 14, priv: 0, post_type: 1 })" data-controller="s-tooltip " data-s-tooltip-placement="bottom" data-s-popover-placement="bottom" aria-controls="" title="Follow this question to receive notifications"> Follow </button> </div> </div> <div class="js-menu-popup-container"></div> </div> </div> <div class="post-signature owner flex--item"> <div class="user-info "> <div class="user-action-time"> asked <span title="2023-03-05 19:15:18Z" class="relativetime">12 hours ago</span> </div> <div class="user-gravatar32"> <a href="/web/20230306075114/https://diy.stackexchange.com/users/150708/sourmonk"><div class="gravatar-wrapper-32"><img src="https://web.archive.org/web/20230306075114im_/https://www.gravatar.com/avatar/caef50bfbb9cf48f704e58592bcfcf5c?s=64&amp;d=identicon&amp;r=PG&amp;f=1" alt="SourMonk's user avatar" width="32" height="32" class="bar-sm"></div></a> </div> <div class="user-details" itemprop="author" itemscope itemtype="http://schema.org/Person"> <a href="/web/20230306075114/https://diy.stackexchange.com/users/150708/sourmonk">SourMonk</a><span class="d-none" itemprop="name">SourMonk</span> <div class="-flair"> <span class="reputation-score" title="reputation score " dir="ltr">131</span><span title="2 bronze badges" aria-hidden="true"><span class="badge3"></span><span class="badgecount">2</span></span><span class="v-visible-sr">2 bronze badges</span> </div> </div> </div> <div class="js-new-contributor-indicator ps-relative"> <div class="new-contributor-indicator fc-medium py6 px8 bbr-sm"> <span class="js-new-contributor-label"><svg aria-hidden="true" class="svg-icon iconWave" width="18" height="18" viewbox="0 0 18 18"><path fill="#FFC166" d="M10.7 17c-2.3 0-3.9-2.05-5.07-3.54l-.49-.6c-.67-.8-1.59-1.63-2.4-2.36A10.91 10.91 0 0 1 1.1 8.87c-.16-.28-.15-.8.18-.96.44-.23.95-.05 1.32.24l3.5 2.75c.1.08.2-.06.13-.15-.57-.8-3.42-4.77-3.71-5.15-.49-.63-.17-1.45.68-1.45.44 0 .76.28.96.51l3.6 4.42c.08.09.23 0 .17-.1L4.41 2.96c-.64-1.1 1.13-2 1.77-.9l3.8 6.13c.07.1.22.03.18-.09A153.8 153.8 0 0 0 8.1 2.54c-.31-.68-.2-1.14.36-1.42.52-.27 1.14-.07 1.47.48l3.69 8.3c.02.04.05.05.1.06a.1.1 0 0 0 .09-.07l.66-2.28c.2-.66.95-1 1.6-.81.7.18 1.09.86.89 1.5-.33 1.27-.7 2.52-1.09 3.77-.58 1.86-1.03 3.33-3.11 4.4-.7.35-1.38.53-2.05.53Z"/><path d="M14 .37a.5.5 0 0 0-.88.45l1.96 3.9a.5.5 0 0 0 .9-.45L14 .37Zm-1.17 2.17a.5.5 0 0 0-.91.42l.84 1.87a.5.5 0 1 0 .91-.41l-.84-1.88Zm-10.6 9.74a.5.5 0 0 1 .7-.02l1.7 1.6a.5.5 0 0 1-.7.72l-1.68-1.6a.5.5 0 0 1-.02-.7Zm-1.39.98a.5.5 0 1 0-.68.73l3.03 2.84a.5.5 0 0 0 .68-.73L.84 13.26Z" opacity=".4"/></svg> New contributor</span> </div> <div class="js-new-contributor-popover s-popover fs-caption"> <div class="s-popover--arrow s-popover--arrow__tc"></div> <a href="/web/20230306075114/https://diy.stackexchange.com/users/150708/sourmonk">SourMonk</a> is a new contributor to this site. Take care in asking for clarification, commenting, and answering. Check out our <a href="/web/20230306075114/https://diy.stackexchange.com/conduct">Code of Conduct</a>. </div> </div> </div> </div> </div> </div> <span class="d-none" itemprop="commentCount"></span> <div class="post-layout--right js-post-comments-component"> <div id="comments-268276" class="comments js-comments-container bt bc-black-075 mt12 dno" data-post-id="268276" data-min-length="15"> <ul class="comments-list js-comments-list" data-remaining-comments-count="0" data-canpost="false" data-cansee="true" data-comments-unavailable="false" data-addlink-disabled="true"> </ul> </div> <div id="comments-link-268276" data-rep="50" data-anon="true"> <a class="js-add-link comments-link disabled-link" title="Use comments to ask for more information or suggest improvements. Avoid answering questions in comments." href="#" role="button">Add a comment</a> <span class="js-link-separator dno">&nbsp;|&nbsp;</span> <a class="js-show-link comments-link dno" title="Expand to show all comments on this post" href="#" onclick="" role="button"></a> </div> </div> </div> </div> <div id="answers"> <a name="tab-top"></a> <div id="answers-header"> <div class="answers-subheader d-flex ai-center mb8"> <div class="flex--item fl1"> <h2 class="mb0" data-answercount="2"> 2 Answers <span style="display:none;" itemprop="answerCount">2</span> </h2> </div> <div class="flex--item"> <div class="d-flex g4 gsx ai-center sm:fd-column sm:ai-start"> <div class="d-flex fd-column ai-end sm:ai-start"> <label class="flex--item fs-caption" for="answer-sort-dropdown-select-menu"> Sorted by: </label> <a class="js-sort-preference-change s-link flex--item fs-fine d-none" data-value="ScoreDesc" href="/web/20230306075114/https://diy.stackexchange.com/questions/268276/what-kind-of-wall-is-this-and-how-can-i-repair-it?answertab=scoredesc#tab-top"> Reset to default </a> </div> <div class="flex--item s-select"> <select id="answer-sort-dropdown-select-menu"> <option value="scoredesc" selected="selected"> Highest score (default) </option> <option value="modifieddesc"> Date modified (newest first) </option> <option value="createdasc"> Date created (oldest first) </option> </select> </div> </div> </div> </div> </div> <a name="268278"></a> <div id="answer-268278" class="answer js-answer" data-answerid="268278" data-parentid="268276" data-score="4" data-position-on-page="1" data-highest-scored="1" data-question-has-accepted-highest-score="0" itemprop="suggestedAnswer" itemscope itemtype="https://schema.org/Answer"> <div class="post-layout"> <div class="votecell post-layout--left"> <div class="js-voting-container d-flex jc-center fd-column ai-stretch gs4 fc-black-200" data-post-id="268278"> <button class="js-vote-up-btn flex--item s-btn s-btn__unset c-pointer " data-controller="s-tooltip" data-s-tooltip-placement="right" title="This answer is useful" aria-pressed="false" aria-label="Up vote" data-selected-classes="fc-theme-primary" data-unselected-classes=""> <svg aria-hidden="true" class="svg-icon iconArrowUpLg" width="36" height="36" viewbox="0 0 36 36"><path d="M2 25h32L18 9 2 25Z"/></svg> </button> <div class="js-vote-count flex--item d-flex fd-column ai-center fc-black-500 fs-title" itemprop="upvoteCount" data-value="4"> 4 </div> <button class="js-vote-down-btn flex--item s-btn s-btn__unset c-pointer " data-controller="s-tooltip" data-s-tooltip-placement="right" title="This answer is not useful" aria-pressed="false" aria-label="Down vote" data-selected-classes="fc-theme-primary" data-unselected-classes=""> <svg aria-hidden="true" class="svg-icon iconArrowDownLg" width="36" height="36" viewbox="0 0 36 36"><path d="M2 11h32L18 27 2 11Z"/></svg> </button> <button class="js-saves-btn s-btn s-btn__unset c-pointer py4" id="saves-btn-268278" data-controller="s-tooltip" data-s-tooltip-placement="right" data-s-popover-placement="" title="Save this answer." aria-pressed="false" data-post-id="268278" data-post-type-id="2" data-user-privilege-for-post-click="0" aria-controls="" data-s-popover-auto-show="false"> <svg aria-hidden="true" class="fc-theme-primary-500 js-saves-btn-selected d-none svg-icon iconBookmark" width="18" height="18" viewbox="0 0 18 18"><path d="M3 17V3c0-1.1.9-2 2-2h8a2 2 0 0 1 2 2v14l-6-4-6 4Z"/></svg> <svg aria-hidden="true" class="js-saves-btn-unselected svg-icon iconBookmarkAlt" width="18" height="18" viewbox="0 0 18 18"><path d="m9 10.6 4 2.66V3H5v10.26l4-2.66ZM3 17V3c0-1.1.9-2 2-2h8a2 2 0 0 1 2 2v14l-6-4-6 4Z"/></svg> </button> <div class="js-accepted-answer-indicator flex--item fc-green-700 py6 mtn8 d-none" data-s-tooltip-placement="right" title="Loading when this answer was accepted…" tabindex="0" role="note" aria-label="Accepted"> <div class="ta-center"> <svg aria-hidden="true" class="svg-icon iconCheckmarkLg" width="36" height="36" viewbox="0 0 36 36"><path d="m6 14 8 8L30 6v8L14 30l-8-8v-8Z"/></svg> </div> </div> <a class="js-post-issue flex--item s-btn s-btn__unset c-pointer py6 mx-auto" href="/web/20230306075114/https://diy.stackexchange.com/posts/268278/timeline" data-shortcut="T" data-ks-title="timeline" data-controller="s-tooltip" data-s-tooltip-placement="right" title="Show activity on this post." aria-label="Timeline"><svg aria-hidden="true" class="mln2 mr0 svg-icon iconHistory" width="19" height="18" viewbox="0 0 19 18"><path d="M3 9a8 8 0 1 1 3.73 6.77L8.2 14.3A6 6 0 1 0 5 9l3.01-.01-4 4-4-4h3L3 9Zm7-4h1.01L11 9.36l3.22 2.1-.6.93L10 10V5Z"/></svg></a> </div> </div> <div class="answercell post-layout--right"> <div class="s-prose js-post-body" itemprop="text"> <p>Based on the picture, plaster at the back, covered with hardboard (unless it's thinner than it looks and just wallpaper) with a printed surface, covered with some sort of skimcoat (plaster or drywall compound) (poorly adhered) and paint.</p> <p>You could remove all the loose skimcoat, but that might turn out to be the whole wall; scuff the surface behind for better bonding and recoat. You could go ahead and remove the hardboard layer as well, and repair the plaster behind that. Somewhat a matter of how deep you want to dig into &quot;bad things done to this wall&quot; when trying to make them better.</p> </div> <div class="mt24"> <div class="d-flex fw-wrap ai-start jc-end gs8 gsy"> <time itemprop="dateCreated" datetime="2023-03-05T19:25:13"></time> <div class="flex--item mr16" style="flex: 1 1 100px;"> <div class="js-post-menu pt2" data-post-id="268278" data-post-type-id="2"> <div class="d-flex gs8 s-anchors s-anchors__muted fw-wrap"> <div class="flex--item"> <a href="/web/20230306075114/https://diy.stackexchange.com/a/268278" rel="nofollow" itemprop="url" class="js-share-link js-gps-track" title="Short permalink to this answer" data-gps-track="post.click({ item: 2, priv: 0, post_type: 2 })" data-controller="se-share-sheet" data-se-share-sheet-title="Share a link to this answer" data-se-share-sheet-subtitle="" data-se-share-sheet-post-type="answer" data-se-share-sheet-social="facebook twitter " data-se-share-sheet-location="2" data-se-share-sheet-license-url="https%3a%2f%2fcreativecommons.org%2flicenses%2fby-sa%2f4.0%2f" data-se-share-sheet-license-name="CC BY-SA 4.0" data-s-popover-placement="bottom-start">Share</a> </div> <div class="flex--item"> <a href="/web/20230306075114/https://diy.stackexchange.com/posts/268278/edit" class="js-suggest-edit-post js-gps-track" data-gps-track="post.click({ item: 6, priv: 0, post_type: 2 })" title="">Improve this answer</a> </div> <div class="flex--item"> <button type="button" id="btnFollowPost-268278" class="s-btn s-btn__link js-follow-post js-follow-answer js-gps-track" data-gps-track="post.click({ item: 14, priv: 0, post_type: 2 })" data-controller="s-tooltip " data-s-tooltip-placement="bottom" data-s-popover-placement="bottom" aria-controls="" title="Follow this answer to receive notifications"> Follow </button> </div> </div> <div class="js-menu-popup-container"></div> </div> </div> <div class="post-signature flex--item fl0"> <div class="user-info "> <div class="user-action-time"> answered <span title="2023-03-05 19:25:13Z" class="relativetime">12 hours ago</span> </div> <div class="user-gravatar32"> <a href="/web/20230306075114/https://diy.stackexchange.com/users/18078/ecnerwal"><div class="gravatar-wrapper-32"><img src="https://web.archive.org/web/20230306075114im_/https://i.stack.imgur.com/bhld0.jpg?s=64&amp;g=1" alt="Ecnerwal's user avatar" width="32" height="32" class="bar-sm"></div></a> </div> <div class="user-details" itemprop="author" itemscope itemtype="http://schema.org/Person"> <a href="/web/20230306075114/https://diy.stackexchange.com/users/18078/ecnerwal">Ecnerwal</a><span class="d-none" itemprop="name">Ecnerwal</span> <div class="-flair"> <span class="reputation-score" title="reputation score 163,570" dir="ltr">164k</span><span title="9 gold badges" aria-hidden="true"><span class="badge1"></span><span class="badgecount">9</span></span><span class="v-visible-sr">9 gold badges</span><span title="196 silver badges" aria-hidden="true"><span class="badge2"></span><span class="badgecount">196</span></span><span class="v-visible-sr">196 silver badges</span><span title="407 bronze badges" aria-hidden="true"><span class="badge3"></span><span class="badgecount">407</span></span><span class="v-visible-sr">407 bronze badges</span> </div> </div> </div> </div> </div> </div> </div> <span class="d-none" itemprop="commentCount">1</span> <div class="post-layout--right js-post-comments-component"> <div id="comments-268278" class="comments js-comments-container bt bc-black-075 mt12 " data-post-id="268278" data-min-length="15"> <ul class="comments-list js-comments-list" data-remaining-comments-count="0" data-canpost="false" data-cansee="true" data-comments-unavailable="false" data-addlink-disabled="true"> <li id="comment-549006" class="comment js-comment " data-comment-id="549006" data-comment-owner-id="110329" data-comment-score="0"> <div class="js-comment-actions comment-actions"> <div class="comment-score js-comment-score js-comment-edit-hide"> </div> </div> <div class="comment-text js-comment-text-and-form"> <div class="comment-body js-comment-edit-hide"> <span class="comment-copy">Looks like plaster on the outermost layer, to me, given it white color and the way it&#39;s cracking.</span> <div class="d-inline-flex ai-center"> &ndash;&nbsp;<a href="/web/20230306075114/https://diy.stackexchange.com/users/110329/stevesh" title="5,712 reputation" class="comment-user">SteveSh</a> </div> <span class="comment-date" dir="ltr"><span title="2023-03-05 19:31:42Z, License: CC BY-SA 4.0" class="relativetime-clean">12 hours ago</span></span> </div> </div> </li> </ul> </div> <div id="comments-link-268278" data-rep="50" data-anon="true"> <a class="js-add-link comments-link disabled-link" title="Use comments to ask for more information or suggest improvements. Avoid comments like “+1” or “thanks”." href="#" role="button">Add a comment</a> <span class="js-link-separator dno">&nbsp;|&nbsp;</span> <a class="js-show-link comments-link dno" title="Expand to show all comments on this post" href="#" onclick="" role="button"></a> </div> </div> </div> </div> <div class="js-zone-container zone-container-main"> <div id="dfp-tlb" class="everyonelovesstackoverflow everyoneloves__top-leaderboard everyoneloves__leaderboard"></div> <div class="js-report-ad-button-container " style="width: 728px"></div> </div> <a name="268309"></a> <div id="answer-268309" class="answer js-answer" data-answerid="268309" data-parentid="268276" data-score="1" data-position-on-page="2" data-highest-scored="0" data-question-has-accepted-highest-score="0" itemprop="suggestedAnswer" itemscope itemtype="https://schema.org/Answer"> <div class="post-layout"> <div class="votecell post-layout--left"> <div class="js-voting-container d-flex jc-center fd-column ai-stretch gs4 fc-black-200" data-post-id="268309"> <button class="js-vote-up-btn flex--item s-btn s-btn__unset c-pointer " data-controller="s-tooltip" data-s-tooltip-placement="right" title="This answer is useful" aria-pressed="false" aria-label="Up vote" data-selected-classes="fc-theme-primary" data-unselected-classes=""> <svg aria-hidden="true" class="svg-icon iconArrowUpLg" width="36" height="36" viewbox="0 0 36 36"><path d="M2 25h32L18 9 2 25Z"/></svg> </button> <div class="js-vote-count flex--item d-flex fd-column ai-center fc-black-500 fs-title" itemprop="upvoteCount" data-value="1"> 1 </div> <button class="js-vote-down-btn flex--item s-btn s-btn__unset c-pointer " data-controller="s-tooltip" data-s-tooltip-placement="right" title="This answer is not useful" aria-pressed="false" aria-label="Down vote" data-selected-classes="fc-theme-primary" data-unselected-classes=""> <svg aria-hidden="true" class="svg-icon iconArrowDownLg" width="36" height="36" viewbox="0 0 36 36"><path d="M2 11h32L18 27 2 11Z"/></svg> </button> <button class="js-saves-btn s-btn s-btn__unset c-pointer py4" id="saves-btn-268309" data-controller="s-tooltip" data-s-tooltip-placement="right" data-s-popover-placement="" title="Save this answer." aria-pressed="false" data-post-id="268309" data-post-type-id="2" data-user-privilege-for-post-click="0" aria-controls="" data-s-popover-auto-show="false"> <svg aria-hidden="true" class="fc-theme-primary-500 js-saves-btn-selected d-none svg-icon iconBookmark" width="18" height="18" viewbox="0 0 18 18"><path d="M3 17V3c0-1.1.9-2 2-2h8a2 2 0 0 1 2 2v14l-6-4-6 4Z"/></svg> <svg aria-hidden="true" class="js-saves-btn-unselected svg-icon iconBookmarkAlt" width="18" height="18" viewbox="0 0 18 18"><path d="m9 10.6 4 2.66V3H5v10.26l4-2.66ZM3 17V3c0-1.1.9-2 2-2h8a2 2 0 0 1 2 2v14l-6-4-6 4Z"/></svg> </button> <div class="js-accepted-answer-indicator flex--item fc-green-700 py6 mtn8 d-none" data-s-tooltip-placement="right" title="Loading when this answer was accepted…" tabindex="0" role="note" aria-label="Accepted"> <div class="ta-center"> <svg aria-hidden="true" class="svg-icon iconCheckmarkLg" width="36" height="36" viewbox="0 0 36 36"><path d="m6 14 8 8L30 6v8L14 30l-8-8v-8Z"/></svg> </div> </div> <a class="js-post-issue flex--item s-btn s-btn__unset c-pointer py6 mx-auto" href="/web/20230306075114/https://diy.stackexchange.com/posts/268309/timeline" data-shortcut="T" data-ks-title="timeline" data-controller="s-tooltip" data-s-tooltip-placement="right" title="Show activity on this post." aria-label="Timeline"><svg aria-hidden="true" class="mln2 mr0 svg-icon iconHistory" width="19" height="18" viewbox="0 0 19 18"><path d="M3 9a8 8 0 1 1 3.73 6.77L8.2 14.3A6 6 0 1 0 5 9l3.01-.01-4 4-4-4h3L3 9Zm7-4h1.01L11 9.36l3.22 2.1-.6.93L10 10V5Z"/></svg></a> </div> </div> <div class="answercell post-layout--right"> <div class="s-prose js-post-body" itemprop="text"> <p>Honestly, I’d get a real drywall guy in to do an old world finish. Paint matte or slightly more shiny and it’ll look awesome.</p> <p>Alternatives include stripping everything (including ancient trim, which doesn’t like being stripped for re-use, plus budget for lead remediation) and applying new drywall. (If you strip the plaster/lath as well, you’ll probably find that your studs need a lot of massaging to be plumb, straight, and in plane with each other. (Hint: $$$)</p> <p>Or spot patch regularly for the foreseeable future, because poorly bonded mud/ plaster/ whatever that is will keep cracking over time.</p> </div> <div class="mt24"> <div class="d-flex fw-wrap ai-start jc-end gs8 gsy"> <time itemprop="dateCreated" datetime="2023-03-05T23:50:55"></time> <div class="flex--item mr16" style="flex: 1 1 100px;"> <div class="js-post-menu pt2" data-post-id="268309" data-post-type-id="2"> <div class="d-flex gs8 s-anchors s-anchors__muted fw-wrap"> <div class="flex--item"> <a href="/web/20230306075114/https://diy.stackexchange.com/a/268309" rel="nofollow" itemprop="url" class="js-share-link js-gps-track" title="Short permalink to this answer" data-gps-track="post.click({ item: 2, priv: 0, post_type: 2 })" data-controller="se-share-sheet" data-se-share-sheet-title="Share a link to this answer" data-se-share-sheet-subtitle="" data-se-share-sheet-post-type="answer" data-se-share-sheet-social="facebook twitter " data-se-share-sheet-location="2" data-se-share-sheet-license-url="https%3a%2f%2fcreativecommons.org%2flicenses%2fby-sa%2f4.0%2f" data-se-share-sheet-license-name="CC BY-SA 4.0" data-s-popover-placement="bottom-start">Share</a> </div> <div class="flex--item"> <a href="/web/20230306075114/https://diy.stackexchange.com/posts/268309/edit" class="js-suggest-edit-post js-gps-track" data-gps-track="post.click({ item: 6, priv: 0, post_type: 2 })" title="">Improve this answer</a> </div> <div class="flex--item"> <button type="button" id="btnFollowPost-268309" class="s-btn s-btn__link js-follow-post js-follow-answer js-gps-track" data-gps-track="post.click({ item: 14, priv: 0, post_type: 2 })" data-controller="s-tooltip " data-s-tooltip-placement="bottom" data-s-popover-placement="bottom" aria-controls="" title="Follow this answer to receive notifications"> Follow </button> </div> </div> <div class="js-menu-popup-container"></div> </div> </div> <div class="post-signature flex--item fl0"> <div class="user-info user-hover"> <div class="user-action-time"> answered <span title="2023-03-05 23:50:55Z" class="relativetime">8 hours ago</span> </div> <div class="user-gravatar32"> <a href="/web/20230306075114/https://diy.stackexchange.com/users/32284/aloysius-defenestrate"><div class="gravatar-wrapper-32"><img src="https://web.archive.org/web/20230306075114im_/https://www.gravatar.com/avatar/003ea0e428de29d1318f95ea336b13e3?s=64&amp;d=identicon&amp;r=PG&amp;f=1" alt="Aloysius Defenestrate's user avatar" width="32" height="32" class="bar-sm"></div></a> </div> <div class="user-details" itemprop="author" itemscope itemtype="http://schema.org/Person"> <a href="/web/20230306075114/https://diy.stackexchange.com/users/32284/aloysius-defenestrate">Aloysius Defenestrate</a><span class="d-none" itemprop="name">Aloysius Defenestrate</span> <div class="-flair"> <span class="reputation-score" title="reputation score 17,737" dir="ltr">17.7k</span><span title="1 gold badge" aria-hidden="true"><span class="badge1"></span><span class="badgecount">1</span></span><span class="v-visible-sr">1 gold badge</span><span title="30 silver badges" aria-hidden="true"><span class="badge2"></span><span class="badgecount">30</span></span><span class="v-visible-sr">30 silver badges</span><span title="52 bronze badges" aria-hidden="true"><span class="badge3"></span><span class="badgecount">52</span></span><span class="v-visible-sr">52 bronze badges</span> </div> </div> </div> </div> </div> </div> </div> <span class="d-none" itemprop="commentCount"></span> <div class="post-layout--right js-post-comments-component"> <div id="comments-268309" class="comments js-comments-container bt bc-black-075 mt12 dno" data-post-id="268309" data-min-length="15"> <ul class="comments-list js-comments-list" data-remaining-comments-count="0" data-canpost="false" data-cansee="true" data-comments-unavailable="false" data-addlink-disabled="true"> </ul> </div> <div id="comments-link-268309" data-rep="50" data-anon="true"> <a class="js-add-link comments-link disabled-link" title="Use comments to ask for more information or suggest improvements. Avoid comments like “+1” or “thanks”." href="#" role="button">Add a comment</a> <span class="js-link-separator dno">&nbsp;|&nbsp;</span> <a class="js-show-link comments-link dno" title="Expand to show all comments on this post" href="#" onclick="" role="button"></a> </div> </div> </div> </div> <a name="new-answer"></a> <form id="post-form" action="/web/20230306075114/https://diy.stackexchange.com/questions/268276/answer/submit" method="post" class="js-add-answer-component post-form"> <input type="hidden" id="post-id" value="268276"/> <input type="hidden" id="qualityBanWarningShown" name="qualityBanWarningShown" value="false"/> <input type="hidden" name="referrer" value=""/> <h2 class="space" id="your-answer-header"> Your Answer </h2> <script> StackExchange.ready(function() { var channelOptions = { tags: "".split(" "), id: "73" }; initTagRenderer("".split(" "), "".split(" "), channelOptions); StackExchange.using("externalEditor", function() { // Have to fire editor after snippets, if snippets enabled if (StackExchange.settings.snippets.snippetsEnabled) { StackExchange.using("snippets", function() { createEditor(); }); } else { createEditor(); } }); function createEditor() { StackExchange.prepareEditor({ useStacksEditor: false, heartbeatType: 'answer', autoActivateHeartbeat: false, convertImagesToLinks: false, noModals: true, showLowRepImageUploadWarning: true, reputationToPostImages: null, bindNavPrevention: true, postfix: "", imageUploader: { brandingHtml: "Powered by \u003ca href=\"https://imgur.com/\"\u003e\u003csvg class=\"svg-icon\" width=\"50\" height=\"18\" viewBox=\"0 0 50 18\" fill=\"none\" xmlns=\"http://www.w3.org/2000/svg\"\u003e\u003ctitle\u003eImgur Logo\u003c/title\u003e\u003cpath d=\"M46.1709 9.17788C46.1709 8.26454 46.2665 7.94324 47.1084 7.58816C47.4091 7.46349 47.7169 7.36433 48.0099 7.26993C48.9099 6.97997 49.672 6.73443 49.672 5.93063C49.672 5.22043 48.9832 4.61182 48.1414 4.61182C47.4335 4.61182 46.7256 4.91628 46.0943 5.50789C45.7307 4.9328 45.2525 4.66231 44.6595 4.66231C43.6264 4.66231 43.1481 5.28821 43.1481 6.59048V11.9512C43.1481 13.2535 43.6264 13.8962 44.6595 13.8962C45.6924 13.8962 46.1709 13.2535 46.1709 11.9512V9.17788Z\"/\u003e\u003cpath d=\"M32.492 10.1419C32.492 12.6954 34.1182 14.0484 37.0451 14.0484C39.9723 14.0484 41.5985 12.6954 41.5985 10.1419V6.59049C41.5985 5.28821 41.1394 4.66232 40.1061 4.66232C39.0732 4.66232 38.5948 5.28821 38.5948 6.59049V9.60062C38.5948 10.8521 38.2696 11.5455 37.0451 11.5455C35.8209 11.5455 35.4954 10.8521 35.4954 9.60062V6.59049C35.4954 5.28821 35.0173 4.66232 34.0034 4.66232C32.9703 4.66232 32.492 5.28821 32.492 6.59049V10.1419Z\" /\u003e\u003cpath fill-rule=\"evenodd\" clip-rule=\"evenodd\" d=\"M25.6622 17.6335C27.8049 17.6335 29.3739 16.9402 30.2537 15.6379C30.8468 14.7755 30.9615 13.5579 30.9615 11.9512V6.59049C30.9615 5.28821 30.4833 4.66231 29.4502 4.66231C28.9913 4.66231 28.4555 4.94978 28.1109 5.50789C27.499 4.86533 26.7335 4.56087 25.7005 4.56087C23.1369 4.56087 21.0134 6.57349 21.0134 9.27932C21.0134 11.9852 23.003 13.913 25.3754 13.913C26.5612 13.913 27.4607 13.4902 28.1109 12.6616C28.1109 12.7229 28.1161 12.7799 28.121 12.8346C28.1256 12.8854 28.1301 12.9342 28.1301 12.983C28.1301 14.4373 27.2502 15.2321 25.777 15.2321C24.8349 15.2321 24.1352 14.9821 23.5661 14.7787C23.176 14.6393 22.8472 14.5218 22.5437 14.5218C21.7977 14.5218 21.2429 15.0123 21.2429 15.6887C21.2429 16.7375 22.9072 17.6335 25.6622 17.6335ZM24.1317 9.27932C24.1317 7.94324 24.9928 7.09766 26.1024 7.09766C27.2119 7.09766 28.0918 7.94324 28.0918 9.27932C28.0918 10.6321 27.2311 11.5116 26.1024 11.5116C24.9737 11.5116 24.1317 10.6491 24.1317 9.27932Z\"/\u003e\u003cpath d=\"M16.8045 11.9512C16.8045 13.2535 17.2637 13.8962 18.2965 13.8962C19.3298 13.8962 19.8079 13.2535 19.8079 11.9512V8.12928C19.8079 5.82936 18.4879 4.62866 16.4027 4.62866C15.1594 4.62866 14.279 4.98375 13.3609 5.88013C12.653 5.05154 11.6581 4.62866 10.3573 4.62866C9.34336 4.62866 8.57809 4.89931 7.9466 5.5079C7.58314 4.9328 7.10506 4.66232 6.51203 4.66232C5.47873 4.66232 5.00066 5.28821 5.00066 6.59049V11.9512C5.00066 13.2535 5.47873 13.8962 6.51203 13.8962C7.54479 13.8962 8.0232 13.2535 8.0232 11.9512V8.90741C8.0232 7.58817 8.44431 6.91179 9.53458 6.91179C10.5104 6.91179 10.893 7.58817 10.893 8.94108V11.9512C10.893 13.2535 11.3711 13.8962 12.4044 13.8962C13.4375 13.8962 13.9157 13.2535 13.9157 11.9512V8.90741C13.9157 7.58817 14.3365 6.91179 15.4269 6.91179C16.4027 6.91179 16.8045 7.58817 16.8045 8.94108V11.9512Z\"/\u003e\u003cpath d=\"M3.31675 6.59049C3.31675 5.28821 2.83866 4.66232 1.82471 4.66232C0.791758 4.66232 0.313354 5.28821 0.313354 6.59049V11.9512C0.313354 13.2535 0.791758 13.8962 1.82471 13.8962C2.85798 13.8962 3.31675 13.2535 3.31675 11.9512V6.59049Z\" /\u003e\u003cpath d=\"M1.87209 0.400291C0.843612 0.400291 0 1.1159 0 1.98861C0 2.87869 0.822846 3.57676 1.87209 3.57676C2.90056 3.57676 3.7234 2.87869 3.7234 1.98861C3.7234 1.1159 2.90056 0.400291 1.87209 0.400291Z\" fill=\"#1BB76E\"/\u003e\u003c/svg\u003e\u003c/a\u003e", contentPolicyHtml: "User contributions licensed under \u003ca href=\"https://stackoverflow.com/help/licensing\"\u003eCC BY-SA\u003c/a\u003e \u003ca href=\"https://stackoverflow.com/legal/content-policy\"\u003e(content policy)\u003c/a\u003e", allowUrls: true }, noCode: true, onDemand: true, discardSelector: ".discard-answer", enableTables: true, isStacksEditorPreviewEnabled: false ,immediatelyShowMarkdownHelp:true,enableTables:true }); } }); </script> <div id="post-editor" class="post-editor js-post-editor"> <div class="ps-relative"> <div class="wmd-container mb8"> <div id="wmd-button-bar" class="wmd-button-bar btr-sm"></div> <div class="new-contributor-indicator fc-black py12 ta-center bl br bc-black-200"> <svg aria-hidden="true" class="svg-icon iconWave" width="18" height="18" viewbox="0 0 18 18"><path fill="#FFC166" d="M10.7 17c-2.3 0-3.9-2.05-5.07-3.54l-.49-.6c-.67-.8-1.59-1.63-2.4-2.36A10.91 10.91 0 0 1 1.1 8.87c-.16-.28-.15-.8.18-.96.44-.23.95-.05 1.32.24l3.5 2.75c.1.08.2-.06.13-.15-.57-.8-3.42-4.77-3.71-5.15-.49-.63-.17-1.45.68-1.45.44 0 .76.28.96.51l3.6 4.42c.08.09.23 0 .17-.1L4.41 2.96c-.64-1.1 1.13-2 1.77-.9l3.8 6.13c.07.1.22.03.18-.09A153.8 153.8 0 0 0 8.1 2.54c-.31-.68-.2-1.14.36-1.42.52-.27 1.14-.07 1.47.48l3.69 8.3c.02.04.05.05.1.06a.1.1 0 0 0 .09-.07l.66-2.28c.2-.66.95-1 1.6-.81.7.18 1.09.86.89 1.5-.33 1.27-.7 2.52-1.09 3.77-.58 1.86-1.03 3.33-3.11 4.4-.7.35-1.38.53-2.05.53Z"/><path d="M14 .37a.5.5 0 0 0-.88.45l1.96 3.9a.5.5 0 0 0 .9-.45L14 .37Zm-1.17 2.17a.5.5 0 0 0-.91.42l.84 1.87a.5.5 0 1 0 .91-.41l-.84-1.88Zm-10.6 9.74a.5.5 0 0 1 .7-.02l1.7 1.6a.5.5 0 0 1-.7.72l-1.68-1.6a.5.5 0 0 1-.02-.7Zm-1.39.98a.5.5 0 1 0-.68.73l3.03 2.84a.5.5 0 0 0 .68-.73L.84 13.26Z" opacity=".4"/></svg> <a href="/web/20230306075114/https://diy.stackexchange.com/users/150708/sourmonk">SourMonk</a> is a new contributor. Be nice, and check out our <a href="/web/20230306075114/https://diy.stackexchange.com/conduct">Code of Conduct</a>. </div> <div class="js-stacks-validation"> <div class="ps-relative"> <textarea id="wmd-input" name="post-text" class="wmd-input s-input bar0 js-post-body-field" data-editor-type="wmd" data-post-type-id="2" cols="92" rows="15" aria-labelledby="your-answer-header" tabindex="101" data-min-length=""></textarea> </div> <div class="s-input-message mt4 d-none js-stacks-validation-message"></div> </div> </div> </div> <aside class="d-flex ai-start jc-space-between js-answer-help s-notice s-notice__warning pb0 pr4 pt4 mb8 d-none" role="status" aria-hidden="true"> <div class="flex--item pt8"> <p>Thanks for contributing an answer to Home Improvement Stack Exchange!</p><ul><li>Please be sure to <em>answer the question</em>. Provide details and share your research!</li></ul><p>But <em>avoid</em> …</p><ul><li>Asking for help, clarification, or responding to other answers.</li><li>Making statements based on opinion; back them up with references or personal experience.</li></ul><p>To learn more, see our <a href="/web/20230306075114/https://diy.stackexchange.com/help/how-to-answer">tips on writing great answers</a>.</p> </div> <button class="flex--item js-answer-help-close-btn s-btn s-btn__muted fc-dark"> <svg aria-hidden="true" class="svg-icon iconClear" width="18" height="18" viewbox="0 0 18 18"><path d="M15 4.41 13.59 3 9 7.59 4.41 3 3 4.41 7.59 9 3 13.59 4.41 15 9 10.41 13.59 15 15 13.59 10.41 9 15 4.41Z"/></svg> </button> </aside> <div> <div id="draft-saved" class="fc-success h24" style="display:none;">Draft saved</div> <div id="draft-discarded" class="fc-error h24" style="display:none;">Draft discarded</div> </div> <div id="wmd-preview" class="s-prose mb16 wmd-preview js-wmd-preview"></div> <div></div> <div class="edit-block"> <input id="fkey" name="fkey" type="hidden" value="2c18411b777ea03baea08fb46c0b7df9d44c6584e5748f81e49d7acfb2f8f921"> <input id="author" name="author" type="text"> </div> </div> <div class="ps-relative"> <div class="form-item dno new-post-login p0 my16"> <div class="d-flex gs16 md:fd-column new-login-form"> <div class="d-flex fd-column w50 md:w-auto gsy gs8 jc-space-between new-login-left"> <h3 class="flex--item fs-title">Sign up or <a id="login-link" href="/web/20230306075114/https://diy.stackexchange.com/users/login?ssrc=question_page&amp;returnurl=https%3a%2f%2fdiy.stackexchange.com%2fquestions%2f268276%2fwhat-kind-of-wall-is-this-and-how-can-i-repair-it%23new-answer">log in</a></h3> <script> StackExchange.ready(function () { StackExchange.helpers.onClickDraftSave('#login-link'); }); </script> <div class="flex--item s-btn s-btn__muted s-btn__outlined s-btn__icon google-login" data-ga="[&quot;sign up&quot;,&quot;Sign Up Started - Google&quot;,&quot;New Post&quot;,null,null]"> <svg aria-hidden="true" class="native svg-icon iconGoogle" width="18" height="18" viewbox="0 0 18 18"><path fill="#4285F4" d="M16.51 8H8.98v3h4.3c-.18 1-.74 1.48-1.6 2.04v2.01h2.6a7.8 7.8 0 0 0 2.38-5.88c0-.57-.05-.66-.15-1.18Z"/><path fill="#34A853" d="M8.98 17c2.16 0 3.97-.72 5.3-1.94l-2.6-2a4.8 4.8 0 0 1-7.18-2.54H1.83v2.07A8 8 0 0 0 8.98 17Z"/><path fill="#FBBC05" d="M4.5 10.52a4.8 4.8 0 0 1 0-3.04V5.41H1.83a8 8 0 0 0 0 7.18l2.67-2.07Z"/><path fill="#EA4335" d="M8.98 4.18c1.17 0 2.23.4 3.06 1.2l2.3-2.3A8 8 0 0 0 1.83 5.4L4.5 7.49a4.77 4.77 0 0 1 4.48-3.3Z"/></svg> Sign up using Google </div> <div class="flex--item s-btn s-btn__muted s-btn__icon facebook-login" data-ga="[&quot;sign up&quot;,&quot;Sign Up Started - Facebook&quot;,&quot;New Post&quot;,null,null]"> <svg aria-hidden="true" class="svg-icon iconFacebook" width="18" height="18" viewbox="0 0 18 18"><path fill="#4167B2" d="M3 1a2 2 0 0 0-2 2v12c0 1.1.9 2 2 2h12a2 2 0 0 0 2-2V3a2 2 0 0 0-2-2H3Zm6.55 16v-6.2H7.46V8.4h2.09V6.61c0-2.07 1.26-3.2 3.1-3.2.88 0 1.64.07 1.87.1v2.16h-1.29c-1 0-1.19.48-1.19 1.18V8.4h2.39l-.31 2.42h-2.08V17h-2.5Z"/></svg> Sign up using Facebook </div> <div class="flex--item s-btn s-btn__muted s-btn__outlined s-btn__icon stackexchange-login" data-ga="[&quot;sign up&quot;,&quot;Sign Up Navigation&quot;,&quot;New Post&quot;,null,null]"> <svg aria-hidden="true" class="native svg-icon iconLogoGlyphXSm" width="18" height="18" viewbox="0 0 18 18"><path d="M14 16v-5h2v7H2v-7h2v5h10Z" fill="#BCBBBB"/><path d="m12.09.72-1.21.9 4.5 6.07 1.22-.9L12.09.71ZM5 15h8v-2H5v2Zm9.15-5.87L8.35 4.3l.96-1.16 5.8 4.83-.96 1.16Zm-7.7-1.47 6.85 3.19.63-1.37-6.85-3.2-.63 1.38Zm6.53 5L5.4 11.39l.38-1.67 7.42 1.48-.22 1.46Z" fill="#F48024"/></svg> Sign up using Email and Password </div> </div> <input type="hidden" name="use-facebook" class="use-facebook" value="false"/> <input type="hidden" name="use-google" class="use-google" value="false"/> <button type="button" class="d-none js-submit-openid">Submit</button> <div class="d-flex gsy gs8 fd-column w50 md:w-auto new-login-right form-item p0"> <h3 class="flex--item fs-title">Post as a guest</h3> <div class="flex--item"> <div class="d-flex gs4 gsy fd-column"> <label class="s-label" for="display-name">Name</label> <div class="d-flex ps-relative"> <input class="s-input" id="display-name" name="display-name" maxlength="30" type="text" value="" tabindex="105" placeholder=""/> </div> </div> </div> <div class="flex--item"> <div class="d-flex gs4 gsy fd-column"> <div class="flex--item"> <div class="d-flex gs2 gsy fd-column"> <label class="flex--item s-label" for="m-address">Email</label> <p class="flex--item s-description">Required, but never shown</p> </div> </div> <div class="d-flex ps-relative"> <input class="s-input js-post-email-field" id="m-address" name="m-address" type="text" value="" size="40" tabindex="106" placeholder=""/> </div> </div> </div> </div> </div> </div> <script> StackExchange.ready( function () { StackExchange.openid.initPostLogin('.new-post-login', 'https%3a%2f%2fdiy.stackexchange.com%2fquestions%2f268276%2fwhat-kind-of-wall-is-this-and-how-can-i-repair-it%23new-answer', 'question_page'); } ); </script> <noscript> <h3 class="flex--item fs-title">Post as a guest</h3> <div class="flex--item"> <div class="d-flex gs4 gsy fd-column"> <label class="s-label" for="display-name">Name</label> <div class="d-flex ps-relative"> <input class="s-input" id="display-name" name="display-name" maxlength="30" type="text" value="" tabindex="105" placeholder=""/> </div> </div> </div> <div class="flex--item"> <div class="d-flex gs4 gsy fd-column"> <div class="flex--item"> <div class="d-flex gs2 gsy fd-column"> <label class="flex--item s-label" for="m-address">Email</label> <p class="flex--item s-description">Required, but never shown</p> </div> </div> <div class="d-flex ps-relative"> <input class="s-input js-post-email-field" id="m-address" name="m-address" type="text" value="" size="40" tabindex="106" placeholder=""/> </div> </div> </div> </noscript> </div> <div class="form-submit clear-both d-flex gsx gs4"> <button id="submit-button" class="flex--item s-btn s-btn__primary s-btn__icon" type="submit" tabindex="120" autocomplete="off"> Post Your Answer </button> <button class="flex--item s-btn s-btn__danger discard-answer dno"> Discard </button> <p class="privacy-policy-agreement"> By clicking “Post Your Answer”, you agree to our <a href="https://web.archive.org/web/20230306075114/https://stackoverflow.com/legal/terms-of-service/public" name="tos" target="_blank" class="-link">terms of service</a>, <a href="https://web.archive.org/web/20230306075114/https://stackoverflow.com/legal/privacy-policy" name="privacy" target="_blank" class="-link">privacy policy</a> and <a href="https://web.archive.org/web/20230306075114/https://stackoverflow.com/legal/cookie-policy" name="cookie" target="_blank" class="-link">cookie policy</a><input type="hidden" name="legalLinksShown" value="1"/> </p> </div> <div class="js-general-error general-error clear-both d-none" aria-live="polite"></div> </form> <h2 class="bottom-notice" data-loc="1"> <div> Not the answer you&#x27;re looking for? Browse other questions tagged <ul class="ml0 list-ls-none js-post-tag-list-wrapper d-inline"><li class="d-inline mr4 js-post-tag-list-item"><a href="/web/20230306075114/https://diy.stackexchange.com/questions/tagged/repair" class="post-tag" title="show questions tagged 'repair'" aria-label="show questions tagged 'repair'" rel="tag" aria-labelledby="tag-repair-tooltip-container">repair</a></li><li class="d-inline mr4 js-post-tag-list-item"><a href="/web/20230306075114/https://diy.stackexchange.com/questions/tagged/walls" class="post-tag" title="show questions tagged 'walls'" aria-label="show questions tagged 'walls'" rel="tag" aria-labelledby="tag-walls-tooltip-container">walls</a></li></ul> or <a href="/web/20230306075114/https://diy.stackexchange.com/questions/ask">ask your own question</a>. </div> </h2> </div> </div> <div id="sidebar" class="show-votes" role="complementary" aria-label="sidebar"> <div class="s-sidebarwidget s-sidebarwidget__yellow s-anchors s-anchors__grayscale mb16" data-tracker="cb=1"> <ul class="d-block p0 m0"> <li class="s-sidebarwidget--header s-sidebarwidget__small-bold-text d-flex fc-black-600 d:fc-black-900 bb bbw1"> Featured on Meta </li> <li class="s-sidebarwidget--item d-flex px16"> <div class="flex--item1 fl-shrink0"> <div class="favicon favicon-stackexchangemeta" title="Meta Stack Exchange"></div> </div> <div class="flex--item wmn0 ow-break-word"> <a href="https://web.archive.org/web/20230306075114/https://meta.stackexchange.com/questions/386727/weve-added-a-necessary-cookies-only-option-to-the-cookie-consent-popup" class="js-gps-track" data-ga="[&quot;community bulletin board&quot;,&quot;Featured on Meta&quot;,&quot;https://meta.stackexchange.com/questions/386727/weve-added-a-necessary-cookies-only-option-to-the-cookie-consent-popup&quot;,null,null]" data-gps-track="communitybulletin.click({ priority: 3, position: 0 })">We&#39;ve added a &quot;Necessary cookies only&quot; option to the cookie consent popup</a> </div> </li> </ul> </div> <div class="js-zone-container zone-container-sidebar"> <div id="dfp-tsb" class="everyonelovesstackoverflow everyoneloves__top-sidebar"></div> <div class="js-report-ad-button-container " style="width: 300px"></div> </div> <div class="module sidebar-related"> <h4 id="h-related">Related</h4> <div class="related js-gps-related-questions" data-tracker="rq=1"> <div class="spacer"> <a href="/web/20230306075114/https://diy.stackexchange.com/q/6874" title="Question score (upvotes - downvotes)"> <div class="answer-votes answered-accepted default">3</div> </a> <a href="/web/20230306075114/https://diy.stackexchange.com/questions/6874/what-is-up-with-this-wall-and-what-is-the-right-way-to-hang-something-on-it" class="question-hyperlink">What is up with this wall, and what is the right way to hang something on it?</a> </div> <div class="spacer"> <a href="/web/20230306075114/https://diy.stackexchange.com/q/9260" title="Question score (upvotes - downvotes)"> <div class="answer-votes answered-accepted default">7</div> </a> <a href="/web/20230306075114/https://diy.stackexchange.com/questions/9260/how-can-i-tell-if-i-have-rock-or-wood-lath-plaster-and-is-there-a-difference-wh" class="question-hyperlink">How can I tell if I have rock or wood lath plaster, and is there a difference when repairing cracks and peeling/flaking?</a> </div> <div class="spacer"> <a href="/web/20230306075114/https://diy.stackexchange.com/q/21993" title="Question score (upvotes - downvotes)"> <div class="answer-votes default">2</div> </a> <a href="/web/20230306075114/https://diy.stackexchange.com/questions/21993/how-can-i-build-shower-niches-into-an-old-plaster-and-lath-wall" class="question-hyperlink">How can I build shower niches into an old plaster and lath wall?</a> </div> <div class="spacer"> <a href="/web/20230306075114/https://diy.stackexchange.com/q/155966" title="Question score (upvotes - downvotes)"> <div class="answer-votes default">0</div> </a> <a href="/web/20230306075114/https://diy.stackexchange.com/questions/155966/what-is-this-construction-material-under-my-drywall" class="question-hyperlink">What is this construction material under my drywall?</a> </div> <div class="spacer"> <a href="/web/20230306075114/https://diy.stackexchange.com/q/172936" title="Question score (upvotes - downvotes)"> <div class="answer-votes answered-accepted default">19</div> </a> <a href="/web/20230306075114/https://diy.stackexchange.com/questions/172936/what-is-this-sticking-out-of-my-wall" class="question-hyperlink">What is this sticking out of my wall?</a> </div> <div class="spacer"> <a href="/web/20230306075114/https://diy.stackexchange.com/q/179472" title="Question score (upvotes - downvotes)"> <div class="answer-votes answered-accepted default">2</div> </a> <a href="/web/20230306075114/https://diy.stackexchange.com/questions/179472/how-do-i-repair-this-rot-insect-damage-on-my-wall-plate" class="question-hyperlink">How do I repair this rot/insect damage on my wall plate?</a> </div> <div class="spacer"> <a href="/web/20230306075114/https://diy.stackexchange.com/q/182451" title="Question score (upvotes - downvotes)"> <div class="answer-votes default">2</div> </a> <a href="/web/20230306075114/https://diy.stackexchange.com/questions/182451/whats-this-metal-thing-in-my-wall-where-the-studs-should-be-and-how-do-i-put-a" class="question-hyperlink">What&#39;s this metal thing in my wall where the studs should be, and how do I put a shelf on it?</a> </div> <div class="spacer"> <a href="/web/20230306075114/https://diy.stackexchange.com/q/187021" title="Question score (upvotes - downvotes)"> <div class="answer-votes answered-accepted default">2</div> </a> <a href="/web/20230306075114/https://diy.stackexchange.com/questions/187021/how-would-you-repair-this-copper-drain-pipe-and-how-much-pipe-would-you-replace" class="question-hyperlink">How would you repair this copper drain pipe, and how much pipe would you replace?</a> </div> <div class="spacer"> <a href="/web/20230306075114/https://diy.stackexchange.com/q/201842" title="Question score (upvotes - downvotes)"> <div class="answer-votes answered-accepted default">1</div> </a> <a href="/web/20230306075114/https://diy.stackexchange.com/questions/201842/how-to-identify-the-wall-type-and-studs-for-drilling" class="question-hyperlink">How to identify the wall type and studs for drilling</a> </div> <div class="spacer"> <a href="/web/20230306075114/https://diy.stackexchange.com/q/210642" title="Question score (upvotes - downvotes)"> <div class="answer-votes answered-accepted default">1</div> </a> <a href="/web/20230306075114/https://diy.stackexchange.com/questions/210642/what-material-is-my-residence-adjoining-common-wall-made-from" class="question-hyperlink">What material is my residence-adjoining common wall made from?</a> </div> </div> </div> <script type="text/javascript"> $(document).ready(function() { $(".js-gps-related-questions .spacer").each(function (i) { $(this).click(function() { fireSidebarRelatedEvent(i + 1); }); }); }); function fireSidebarRelatedEvent(position) { StackExchange.using("gps", function() { StackExchange.gps.track("inline_experiment.related_questions.click", { position: position, location: '1', // sidebar, originQuestionId: 268276, postNumAnswers: 2 }); }); } </script> <div id="hot-network-questions" class="module tex2jax_ignore"> <h4> <a href="https://web.archive.org/web/20230306075114/https://stackexchange.com/questions?tab=hot" class="js-gps-track s-link s-link__inherit" data-gps-track="posts_hot_network.click({ item_type:1, location:11 })"> Hot Network Questions </a> </h4> <ul> <li> <div class="favicon favicon-physics" title="Physics Stack Exchange"></div><a href="https://web.archive.org/web/20230306075114/https://physics.stackexchange.com/questions/754032/wavefunction-of-a-particle-decay" class="js-gps-track question-hyperlink mb0" data-gps-track="site.switch({ item_type:11, target_site:151 }); posts_hot_network.click({ item_type:2, location:11 })"> Wavefunction of a particle decay </a> </li> <li> <div class="favicon favicon-math" title="Mathematics Stack Exchange"></div><a href="https://web.archive.org/web/20230306075114/https://math.stackexchange.com/questions/4652817/my-contour-integrations-seem-to-contradict-the-residue-theorem" class="js-gps-track question-hyperlink mb0" data-gps-track="site.switch({ item_type:11, target_site:69 }); posts_hot_network.click({ item_type:2, location:11 })"> My contour integrations seem to contradict the residue theorem. </a> </li> <li> <div class="favicon favicon-politics" title="Politics Stack Exchange"></div><a href="https://web.archive.org/web/20230306075114/https://politics.stackexchange.com/questions/78578/why-did-ukraine-abstain-from-the-unhrc-vote-on-china" class="js-gps-track question-hyperlink mb0" data-gps-track="site.switch({ item_type:11, target_site:475 }); posts_hot_network.click({ item_type:2, location:11 })"> Why did Ukraine abstain from the UNHRC vote on China? </a> </li> <li> <div class="favicon favicon-aviation" title="Aviation Stack Exchange"></div><a href="https://web.archive.org/web/20230306075114/https://aviation.stackexchange.com/questions/97901/is-it-suspicious-or-odd-to-stand-by-the-gate-of-a-ga-airport-watching-the-planes" class="js-gps-track question-hyperlink mb0" data-gps-track="site.switch({ item_type:11, target_site:528 }); posts_hot_network.click({ item_type:2, location:11 })"> Is it suspicious or odd to stand by the gate of a GA airport watching the planes? </a> </li> <li> <div class="favicon favicon-electronics" title="Electrical Engineering Stack Exchange"></div><a href="https://web.archive.org/web/20230306075114/https://electronics.stackexchange.com/questions/657513/resistance-depending-on-voltage-the-chicken-and-the-egg" class="js-gps-track question-hyperlink mb0" data-gps-track="site.switch({ item_type:11, target_site:135 }); posts_hot_network.click({ item_type:2, location:11 })"> Resistance depending on voltage - the chicken and the egg? </a> </li> <li class="dno js-hidden"> <div class="favicon favicon-space" title="Space Exploration Stack Exchange"></div><a href="https://web.archive.org/web/20230306075114/https://space.stackexchange.com/questions/63023/sending-a-soyuz-ship-interplanetary-a-plausible-option" class="js-gps-track question-hyperlink mb0" data-gps-track="site.switch({ item_type:11, target_site:508 }); posts_hot_network.click({ item_type:2, location:11 })"> Sending a Soyuz ship interplanetary - a plausible option? </a> </li> <li class="dno js-hidden"> <div class="favicon favicon-academia" title="Academia Stack Exchange"></div><a href="https://web.archive.org/web/20230306075114/https://academia.stackexchange.com/questions/194018/what-is-the-difference-between-paper-presentation-and-poster-presentation" class="js-gps-track question-hyperlink mb0" data-gps-track="site.switch({ item_type:11, target_site:415 }); posts_hot_network.click({ item_type:2, location:11 })"> What is the difference between paper presentation and poster presentation? </a> </li> <li class="dno js-hidden"> <div class="favicon favicon-academia" title="Academia Stack Exchange"></div><a href="https://web.archive.org/web/20230306075114/https://academia.stackexchange.com/questions/194042/should-i-ask-why-they-are-interested-in-me-in-an-interview-for-a-faculty-positio" class="js-gps-track question-hyperlink mb0" data-gps-track="site.switch({ item_type:11, target_site:415 }); posts_hot_network.click({ item_type:2, location:11 })"> Should I ask why they are interested in me in an interview for a faculty position? </a> </li> <li class="dno js-hidden"> <div class="favicon favicon-scifi" title="Science Fiction &amp; Fantasy Stack Exchange"></div><a href="https://web.archive.org/web/20230306075114/https://scifi.stackexchange.com/questions/273725/was-kips-defiance-relevant-to-the-galactics-decision" class="js-gps-track question-hyperlink mb0" data-gps-track="site.switch({ item_type:11, target_site:186 }); posts_hot_network.click({ item_type:2, location:11 })"> Was Kip&#x27;s defiance relevant to the Galactics&#x27; decision? </a> </li> <li class="dno js-hidden"> <div class="favicon favicon-gis" title="Geographic Information Systems Stack Exchange"></div><a href="https://web.archive.org/web/20230306075114/https://gis.stackexchange.com/questions/454455/displaying-label-if-field-contains-x-or-y-value-in-qgis" class="js-gps-track question-hyperlink mb0" data-gps-track="site.switch({ item_type:11, target_site:79 }); posts_hot_network.click({ item_type:2, location:11 })"> Displaying label if field contains &#x27;X&#x27; or &#x27;Y&#x27; value in QGIS </a> </li> <li class="dno js-hidden"> <div class="favicon favicon-scifi" title="Science Fiction &amp; Fantasy Stack Exchange"></div><a href="https://web.archive.org/web/20230306075114/https://scifi.stackexchange.com/questions/273714/what-is-this-movie-where-a-woman-is-attacked-and-possessed-by-a-worm" class="js-gps-track question-hyperlink mb0" data-gps-track="site.switch({ item_type:11, target_site:186 }); posts_hot_network.click({ item_type:2, location:11 })"> What is this movie where a woman is attacked and possessed by a worm </a> </li> <li class="dno js-hidden"> <div class="favicon favicon-dsp" title="Signal Processing Stack Exchange"></div><a href="https://web.archive.org/web/20230306075114/https://dsp.stackexchange.com/questions/86955/how-to-sample-a-complex-function" class="js-gps-track question-hyperlink mb0" data-gps-track="site.switch({ item_type:11, target_site:295 }); posts_hot_network.click({ item_type:2, location:11 })"> How to sample a complex function? </a> </li> <li class="dno js-hidden"> <div class="favicon favicon-tex" title="TeX - LaTeX Stack Exchange"></div><a href="https://web.archive.org/web/20230306075114/https://tex.stackexchange.com/questions/678316/align-vertically-2-circuits" class="js-gps-track question-hyperlink mb0" data-gps-track="site.switch({ item_type:11, target_site:85 }); posts_hot_network.click({ item_type:2, location:11 })"> Align vertically 2 circuits </a> </li> <li class="dno js-hidden"> <div class="favicon favicon-hsm" title="History of Science and Mathematics Stack Exchange"></div><a href="https://web.archive.org/web/20230306075114/https://hsm.stackexchange.com/questions/15206/euler-a-baby-on-his-lap-a-cat-on-his-back-that-s-how-he-wrote-his-immortal" class="js-gps-track question-hyperlink mb0" data-gps-track="site.switch({ item_type:11, target_site:587 }); posts_hot_network.click({ item_type:2, location:11 })"> Euler: &#x201C;A baby on his lap, a cat on his back &#x2014; that&#x2019;s how he wrote his immortal works&#x201D; (origin?) </a> </li> <li class="dno js-hidden"> <div class="favicon favicon-mathoverflow" title="MathOverflow"></div><a href="https://web.archive.org/web/20230306075114/https://mathoverflow.net/questions/442114/quotients-of-number-fields-by-certain-prime-powers" class="js-gps-track question-hyperlink mb0" data-gps-track="site.switch({ item_type:11, target_site:504 }); posts_hot_network.click({ item_type:2, location:11 })"> Quotients of number fields by certain prime powers </a> </li> <li class="dno js-hidden"> <div class="favicon favicon-english" title="English Language &amp; Usage Stack Exchange"></div><a href="https://web.archive.org/web/20230306075114/https://english.stackexchange.com/questions/604303/why-are-some-high-schools-called-hospitals" class="js-gps-track question-hyperlink mb0" data-gps-track="site.switch({ item_type:11, target_site:97 }); posts_hot_network.click({ item_type:2, location:11 })"> Why are some high schools called hospitals? </a> </li> <li class="dno js-hidden"> <div class="favicon favicon-ell" title="English Language Learners Stack Exchange"></div><a href="https://web.archive.org/web/20230306075114/https://ell.stackexchange.com/questions/333708/is-or-are-for-two-uncountable-words" class="js-gps-track question-hyperlink mb0" data-gps-track="site.switch({ item_type:11, target_site:481 }); posts_hot_network.click({ item_type:2, location:11 })"> &quot;Is&quot; or &quot;are&quot; for two uncountable words? </a> </li> <li class="dno js-hidden"> <div class="favicon favicon-mathematica" title="Mathematica Stack Exchange"></div><a href="https://web.archive.org/web/20230306075114/https://mathematica.stackexchange.com/questions/281905/finding-the-output-of-the-nested-compoundexpression" class="js-gps-track question-hyperlink mb0" data-gps-track="site.switch({ item_type:11, target_site:387 }); posts_hot_network.click({ item_type:2, location:11 })"> Finding the output of the nested CompoundExpression </a> </li> <li class="dno js-hidden"> <div class="favicon favicon-cooking" title="Seasoned Advice"></div><a href="https://web.archive.org/web/20230306075114/https://cooking.stackexchange.com/questions/123536/are-there-tables-of-wastage-rates-for-different-fruit-and-veg" class="js-gps-track question-hyperlink mb0" data-gps-track="site.switch({ item_type:11, target_site:49 }); posts_hot_network.click({ item_type:2, location:11 })"> Are there tables of wastage rates for different fruit and veg? </a> </li> <li class="dno js-hidden"> <div class="favicon favicon-conlang" title="Constructed Languages Stack Exchange"></div><a href="https://web.archive.org/web/20230306075114/https://conlang.stackexchange.com/questions/1828/is-it-natural-that-different-quantities-with-the-same-physical-dimension-are-dis" class="js-gps-track question-hyperlink mb0" data-gps-track="site.switch({ item_type:11, target_site:692 }); posts_hot_network.click({ item_type:2, location:11 })"> Is it natural that different quantities with the same physical dimension are distinguished by adverbs? </a> </li> <li class="dno js-hidden"> <div class="favicon favicon-crypto" title="Cryptography Stack Exchange"></div><a href="https://web.archive.org/web/20230306075114/https://crypto.stackexchange.com/questions/105509/does-quantum-computers-crack-rsa-and-aes" class="js-gps-track question-hyperlink mb0" data-gps-track="site.switch({ item_type:11, target_site:281 }); posts_hot_network.click({ item_type:2, location:11 })"> Does Quantum Computers crack RSA and AES? </a> </li> <li class="dno js-hidden"> <div class="favicon favicon-chemistry" title="Chemistry Stack Exchange"></div><a href="https://web.archive.org/web/20230306075114/https://chemistry.stackexchange.com/questions/171867/does-high-pressure-reverse-reaction-between-zinc-and-sulfuric-acid" class="js-gps-track question-hyperlink mb0" data-gps-track="site.switch({ item_type:11, target_site:431 }); posts_hot_network.click({ item_type:2, location:11 })"> Does high pressure reverse reaction between zinc and sulfuric acid? </a> </li> <li class="dno js-hidden"> <div class="favicon favicon-worldbuilding" title="Worldbuilding Stack Exchange"></div><a href="https://web.archive.org/web/20230306075114/https://worldbuilding.stackexchange.com/questions/243783/how-or-would-these-mechanical-wings-work" class="js-gps-track question-hyperlink mb0" data-gps-track="site.switch({ item_type:11, target_site:579 }); posts_hot_network.click({ item_type:2, location:11 })"> How or would these mechanical wings work? </a> </li> <li class="dno js-hidden"> <div class="favicon favicon-mathoverflow" title="MathOverflow"></div><a href="https://web.archive.org/web/20230306075114/https://mathoverflow.net/questions/442124/is-there-a-non-constant-function-on-the-sphere-that-diagonalizes-all-rotations-s" class="js-gps-track question-hyperlink mb0" data-gps-track="site.switch({ item_type:11, target_site:504 }); posts_hot_network.click({ item_type:2, location:11 })"> Is there a non-constant function on the sphere that diagonalizes all rotations simultaneously? </a> </li> </ul> <a href="#" class="show-more js-show-more js-gps-track" data-gps-track="posts_hot_network.click({ item_type:3, location:11 })"> more hot questions </a> </div> <div id="feed-link" class="js-feed-link"> <a href="/web/20230306075114/https://diy.stackexchange.com/feeds/question/268276" title="Feed of this question and its answers"> <svg aria-hidden="true" class="fc-orange-400 svg-icon iconRss" width="18" height="18" viewbox="0 0 18 18"><path d="M3 1a2 2 0 0 0-2 2v12c0 1.1.9 2 2 2h12a2 2 0 0 0 2-2V3a2 2 0 0 0-2-2H3Zm0 1.5c6.9 0 12.5 5.6 12.5 12.5H13C13 9.55 8.45 5 3 5V2.5Zm0 5c4.08 0 7.5 3.41 7.5 7.5H8c0-2.72-2.28-5-5-5V7.5Zm0 5c1.36 0 2.5 1.14 2.5 2.5H3v-2.5Z"/></svg> Question feed </a> </div> <aside class="s-modal js-feed-link-modal" tabindex="-1" role="dialog" aria-labelledby="feed-modal-title" aria-describedby="feed-modal-description" aria-hidden="true"> <div class="s-modal--dialog js-modal-dialog wmx4" role="document" data-controller="se-draggable"> <h1 class="s-modal--header fw-bold js-first-tabbable" id="feed-modal-title" data-se-draggable-target="handle" tabindex="0"> Subscribe to RSS </h1> <div class="d-flex gs4 gsy fd-column"> <div class="flex--item"> <label class="d-block s-label c-default" for="feed-url"> Question feed <p class="s-description mt2" id="feed-modal-description">To subscribe to this RSS feed, copy and paste this URL into your RSS reader.</p> </label> </div> <div class="d-flex ps-relative"> <input class="s-input" type="text" name="feed-url" id="feed-url" readonly="readonly" value="https://diy.stackexchange.com/feeds/question/268276"/> <svg aria-hidden="true" class="s-input-icon fc-orange-400 svg-icon iconRss" width="18" height="18" viewbox="0 0 18 18"><path d="M3 1a2 2 0 0 0-2 2v12c0 1.1.9 2 2 2h12a2 2 0 0 0 2-2V3a2 2 0 0 0-2-2H3Zm0 1.5c6.9 0 12.5 5.6 12.5 12.5H13C13 9.55 8.45 5 3 5V2.5Zm0 5c4.08 0 7.5 3.41 7.5 7.5H8c0-2.72-2.28-5-5-5V7.5Zm0 5c1.36 0 2.5 1.14 2.5 2.5H3v-2.5Z"/></svg> </div> </div> <a class="s-modal--close s-btn s-btn__muted js-modal-close js-last-tabbable" href="#" aria-label="Close"> <svg aria-hidden="true" class="svg-icon iconClearSm" width="14" height="14" viewbox="0 0 14 14"><path d="M12 3.41 10.59 2 7 5.59 3.41 2 2 3.41 5.59 7 2 10.59 3.41 12 7 8.41 10.59 12 12 10.59 8.41 7 12 3.41Z"/></svg> </a> </div> </aside> </div> </div> <script>StackExchange.ready(function(){$.get('/posts/268276/ivc/d58c?prg=7fa0d27e-f391-4fed-937f-1d00e54d7c93');});</script> <noscript><div><img src="/web/20230306075114im_/https://diy.stackexchange.com/posts/268276/ivc/d58c?prg=7fa0d27e-f391-4fed-937f-1d00e54d7c93" class="dno" alt="" width="0" height="0"></div></noscript></div> </div> </div> <script type="text/javascript"> var cam = cam || { opt: {} }; var clcGamLoaderOptions = cam || { opt: {} }; var opt = clcGamLoaderOptions.opt; opt.sf = !1; opt.hb = !1; opt.ll = !0; opt.tlb_position = 0; opt.personalization_consent = !1; opt.targeting_consent = !1; opt.performance_consent = !1; opt.targeting = {Registered:['false'],'nontech-ron':['repair','walls'],NumberOfAnswers:['2']}; opt.adReportEnabled = !1; opt.adReportUrl = '/ads/report-ad'; opt.adReportText = 'Report this ad'; opt.adReportFileTypeErrorMessage = 'Please select a PNG or JPG file.'; opt.adReportFileSizeErrorMessage = 'The file must be under 2 MiB.'; opt.adReportErrorText = 'Error uploading ad report.'; opt.adReportThanksText = 'Thanks for your feedback. We’ll review this against our code of conduct and take action if necessary.'; opt.adReportLoginExpiredMessage = 'Your login session has expired, please login and try again.'; opt.adReportLoginErrorMessage = 'An error occurred when loading the report form - please try again'; opt.adReportModalClass = 'js-ad-report'; opt.requestGuid = '7fa0d27e-f391-4fed-937f-1d00e54d7c93'; opt.targeting.TargetingConsent = ['False_Passive']; const urlParams = new URLSearchParams(window.location.search); if (urlParams.has('dfptestads')) { const dfptestads = urlParams.get('dfptestads'); opt.targeting.DfpTestAds = dfptestads; } </script> <script>;(()=>{"use strict";var __webpack_modules__={23:(e,t,o)=>{o.d(t,{Z7:()=>l,eq:()=>r,kG:()=>n});const a=/^\/tags\//.test(location.pathname)||/^\/questions\/tagged\//.test(location.pathname)?"tag-pages":/^\/$/.test(location.pathname)||/^\/home/.test(location.pathname)?"home-page":"question-pages";let s=location.hostname;const i={slots:{lb:[[728,90]],mlb:[[728,90]],smlb:[[728,90]],bmlb:[[728,90]],sb:e=>"dfp-tsb"===e?[[300,250],[300,600]]:[[300,250]],"tag-sponsorship":[[730,135]],"mobile-below-question":[[320,50],[300,250]],msb:[[300,250],[300,600]],"talent-conversion-tracking":[[1,1]],"site-sponsorship":[[230,60]]},ids:{"dfp-tlb":"lb","dfp-mlb":"mlb","dfp-smlb":"smlb","dfp-bmlb":"bmlb","dfp-tsb":"sb","dfp-isb":"sb","dfp-tag":"tag-sponsorship","dfp-msb":"msb","dfp-sspon":"site-sponsorship","dfp-m-aq":"mobile-below-question"},idsToExcludeFromAdReports:["dfp-sspon"]};function n(){return Object.keys(i.ids)}function r(e){return i.idsToExcludeFromAdReports.indexOf(e)<0}function l(e){var t=e.split("_")[0];const o=i.ids[t];let n=i.slots[o];return"function"==typeof n&&(n=n(t)),{path:`/248424177/${s}/${o}/${a}`,sizes:n,zone:o}}},865:(e,t,o)=>{function a(e){return"string"==typeof e?document.getElementById(e):e}function s(e){return!!(e=a(e))&&"none"===getComputedStyle(e).display}function i(e){return!s(e)}function n(e){return!!e}function r(e){return/^\s*$/.test(a(e).innerHTML)}function l(e){const{style:t}=e;t.height=t.maxHeight=t.minHeight="auto",t.display="none"}function d(e){const{style:t}=e;t.height=t.maxHeight=t.minHeight="auto",t.display="none",[].forEach.call(e.children,d)}function c(e){const{style:t}=e;t.height=t.maxHeight=t.minHeight="auto",t.removeProperty("display")}function p(e){const t=document.createElement("script");t.src=e,document.body.appendChild(t)}function g(e){return o=e,(t=[]).push=function(e){return o(),delete this.push,this.push(e)},t;var t,o}function h(e){let t="function"==typeof HTMLTemplateElement;var o=document.createElement(t?"template":"div");return e=e.trim(),o.innerHTML=e,t?o.content.firstChild:o.firstChild}o.d(t,{$Z:()=>c,Bv:()=>h,Gx:()=>p,Nj:()=>a,QZ:()=>g,cf:()=>l,pn:()=>i,wo:()=>d,xb:()=>r,xj:()=>s,yb:()=>n})},763:(__unused_webpack_module,__webpack_exports__,__webpack_require__)=>{__webpack_require__.d(__webpack_exports__,{t:()=>AdReports});var _common_helper__WEBPACK_IMPORTED_MODULE_2__=__webpack_require__(865),_console__WEBPACK_IMPORTED_MODULE_1__=__webpack_require__(276),_ad_units__WEBPACK_IMPORTED_MODULE_0__=__webpack_require__(23);class AdReports{constructor(e,t){if(this.googletag=e,this.cam=t,this.allowedFileTypes=["image/png","image/jpg","image/jpeg"],this.ignoreValidation=!1,_console__WEBPACK_IMPORTED_MODULE_1__.cM("Ad reporting init"),this.cam=t,this.callOnButtonClick=e=>this.onButtonClick(e),this.googletag.pubads().addEventListener("slotRenderEnded",e=>this.handleSlotRendered(e)),Array.isArray(t.slotsRenderedEvents)){_console__WEBPACK_IMPORTED_MODULE_1__.cM("Adding report button to "+t.slotsRenderedEvents.length+" events that have transpired");for(var o=0;o<t.slotsRenderedEvents.length;o++)this.handleSlotRendered(t.slotsRenderedEvents[o])}}handleSlotRendered(e){if(e&&e.slot&&!e.isEmpty&&(e.creativeId||e.lineItemId||!e.isEmpty)){var t=e.slot.getSlotElementId();if(t){var o=document.getElementById(t);if(o)if((0,_ad_units__WEBPACK_IMPORTED_MODULE_0__.eq)(t)){var a=o?.closest(".js-zone-container")?.querySelector(".js-report-ad-button-container");a.append(this.createButton(e)),a.style.height="24px",_console__WEBPACK_IMPORTED_MODULE_1__.cM("Added report button to the bottom of "+t)}else _console__WEBPACK_IMPORTED_MODULE_1__.cM("Not adding report button to the bottom of "+t+": shouldHaveReportButton = false");else _console__WEBPACK_IMPORTED_MODULE_1__.cM("Not adding report button to the bottom of "+t+": resolved invalid adUnit element")}else _console__WEBPACK_IMPORTED_MODULE_1__.cM("Not adding report button to the bottom of element: invalid adUnitElementId")}else _console__WEBPACK_IMPORTED_MODULE_1__.cM("Not adding report button to the bottom of element: invalid SlotRenderEndedEvent")}async onButtonClick(e){e.preventDefault();let t=e.target;const o=t.dataset.modalUrl,a=t.dataset.googleEventData;return await this.loadModal(o,t,a),!1}createButton(e){let t=document.createElement("button");var o=JSON.stringify(e);return t.dataset.googleEventData=o,t.dataset.modalUrl=this.cam.opt.adReportUrl,t.dataset.adUnit=e.slot.getSlotElementId(),t.classList.add("js-report-ad","s-btn","s-btn__link","fs-fine","mt2","float-right"),t.append(document.createTextNode(this.cam.opt.adReportText)),t.removeEventListener("click",this.callOnButtonClick),t.addEventListener("click",this.callOnButtonClick),t}async loadModal(url,$link,googleEventData){try{await window.StackExchange.helpers.loadModal(url,{returnElements:window.$($link)}),this.initForm(googleEventData)}catch(e){var message="",response=e.responseText?eval(`(${e.responseText})`):null;message=response&&response.isLoggedOut?this.cam.opt.adReportLoginExpiredMessage:this.cam.opt.adReportLoginErrorMessage,window.StackExchange.helpers.showToast(message,{type:"danger"})}}removeModal(){window.StackExchange.helpers.closePopups(document.querySelectorAll("."+this.cam.opt.adReportModalClass),"dismiss")}initForm(e,t=!1){this.ignoreValidation=t,this.$form=document.querySelector(".js-ad-report-form"),this.$googleEventData=this.$form.querySelector(".js-json-data"),this.$adReportReasons=this.$form.querySelectorAll(".js-ad-report-reason"),this.$adReportReasonOther=this.$form.querySelector(".js-ad-report-reason-other"),this.$fileUploaderInput=this.$form.querySelector(".js-file-uploader-input"),this.$imageUploader=this.$form.querySelector(".js-image-uploader"),this.$clearImageUpload=this.$form.querySelector(".js-clear-image-upload"),this.$imageUploaderText=this.$form.querySelector(".js-image-uploader-text"),this.$imageUploaderPreview=this.$form.querySelector(".js-image-uploader-preview"),this.$fileErrorMessage=this.$form.querySelector(".js-file-error");const o=this.$form.querySelector(".js-drag-drop-enabled"),a=this.$form.querySelector(".js-drag-drop-disabled");this.$googleEventData.value=e,this.$adReportReasons.forEach((e,t)=>e.addEventListener("change",e=>{this.$adReportReasonOther.classList.toggle("d-none","3"!==e.target.value)})),this.$fileUploaderInput.addEventListener("change",()=>{this.validateFileInput()&&this.updateImagePreview(this.$fileUploaderInput.files)}),this.$clearImageUpload.addEventListener("click",e=>{e.preventDefault(),this.clearImageUpload()});try{this.$fileUploaderInput[0].value="",this.$imageUploader.addEventListener("dragenter dragover dragleave drop",this.preventDefaults),this.$imageUploader.addEventListener("dragenter dragover",this.handleDragStart),this.$imageUploader.addEventListener("dragleave drop",this.handleDragEnd),this.$imageUploader.addEventListener("drop",this.handleDrop)}catch(e){o.classList.add("d-none"),a.classList.remove("d-none")}this.$form.removeEventListener("",this.handleDragEnd),this.$form.addEventListener("submit",async e=>(e.preventDefault(),this.submitForm(),!1))}clearImageUpload(){this.$fileUploaderInput.value="",this.$imageUploaderPreview.setAttribute("src",""),this.$imageUploaderPreview.classList.add("d-none"),this.$clearImageUpload.classList.add("d-none"),this.$imageUploaderText.classList.remove("d-none"),this.$imageUploader.classList.add("p16","ba","bas-dashed","bc-black-100")}preventDefaults(e){e.preventDefault(),e.stopPropagation()}handleDragStart(e){this.$imageUploader.classList.remove("bas-dashed"),this.$imageUploader.classList.add("bas-solid","bc-black-100")}handleDragEnd(e){this.$imageUploader.classList.remove("bas-solid","bc-black-100"),this.$imageUploader.classList.add("bas-dashed")}handleDrop(e){var t=e.originalEvent.dataTransfer.files;FileReader&&t&&1===t.length&&(this.$fileUploaderInput.files=t,this.validateFileInput()&&this.updateImagePreview(t))}setError(e){this.$fileErrorMessage.parentElement.classList.toggle("has-error",e)}updateImagePreview(e){this.$imageUploader.classList.remove("p16","ba","bas-dashed","bc-black-100"),this.$clearImageUpload.classList.remove("d-none"),this.$imageUploaderText.classList.add("d-none");var t=new FileReader;t.onload=e=>{null!=e.target&&(this.$imageUploaderPreview.setAttribute("src",e.target.result),this.$imageUploaderPreview.classList.remove("d-none"))},t.readAsDataURL(e[0])}validateFileInput(){if(this.ignoreValidation)return!0;const e=this.cam.opt.adReportFileTypeErrorMessage,t=this.cam.opt.adReportFileSizeErrorMessage;if(null==this.$fileUploaderInput.files)return!1;var o=this.$fileUploaderInput.files[0];return null==o?(this.setError(!0),!1):this.allowedFileTypes.indexOf(o.type)<0?(this.$fileErrorMessage.textContent=e,this.$fileErrorMessage.classList.remove("d-none"),this.setError(!0),!1):o.size>2097152?(this.$fileErrorMessage.textContent=t,this.$fileErrorMessage.classList.remove("d-none"),this.setError(!0),!1):(this.$fileErrorMessage.classList.add("d-none"),this.setError(!1),!0)}async submitForm(){if(!this.validateFileInput())return!1;this.$form.querySelector("[type=submit]").setAttribute("disabled","true");var e=JSON.parse(this.$googleEventData.value||"{}");e.Reason=parseInt(this.$form.querySelector(".js-ad-report-reason:checked").value,10),e.Description=this.$adReportReasonOther.value,this.$googleEventData.value=JSON.stringify(e);var t=new FormData(this.$form);try{const e=await window.fetch(this.$form.getAttribute("action"),{method:this.$form.getAttribute("method"),body:t,cache:"no-cache"}),a=e.headers.get("content-type")||"",s=await e.text();if(!e.ok)throw new Error("response not valid");if(0===a.indexOf("text/html")){var o=(0,_common_helper__WEBPACK_IMPORTED_MODULE_2__.Bv)(s);const e=o?o.querySelector(".js-modal-content"):null;if(_console__WEBPACK_IMPORTED_MODULE_1__.cM("$popupContent"),_console__WEBPACK_IMPORTED_MODULE_1__.cM(e),!e)throw new Error(`Could not find .js-modal-content in response from ${this.$form.getAttribute("action")}`);document.querySelector(".js-modal-content").replaceWith(e)}else window.StackExchange.helpers.showToast(this.cam.opt.adReportThanksText,{type:"success"}),this.removeModal()}catch(e){window.StackExchange.helpers.showToast(this.cam.opt.adReportErrorText,{type:"danger"})}finally{let e=this.$form.querySelector("[type=submit]");e&&e.removeAttribute("disabled")}}}},276:(e,t,o)=>{function a(...e){}o.d(t,{cM:()=>a})}},__webpack_module_cache__={};function __webpack_require__(e){var t=__webpack_module_cache__[e];if(void 0!==t)return t.exports;var o=__webpack_module_cache__[e]={exports:{}};return __webpack_modules__[e](o,o.exports,__webpack_require__),o.exports}__webpack_require__.d=(e,t)=>{for(var o in t)__webpack_require__.o(t,o)&&!__webpack_require__.o(e,o)&&Object.defineProperty(e,o,{enumerable:!0,get:t[o]})},__webpack_require__.o=(e,t)=>Object.prototype.hasOwnProperty.call(e,t);var __webpack_exports__={};(()=>{var e=__webpack_require__(276),t=(e=>(e[e.Above=0]="Above",e[e.Below=1]="Below",e))(t||{});const o=Object.assign({},{"lib":"https://web.archive.org/web/20230306075114/https://cdn.sstatic.net/clc/js/bundles/gam_loader_script/gam_loader_script.bundle.741.a1bd5bd69766c6c9192d.min.js","style":null,"u":null,"wa":true,"kt":2000,"tto":true,"h":"clc.stackoverflow.com","allowed":"^(((talent\\.)?stackoverflow)|(blog\\.codinghorror)|(.*\\.googlesyndication)|(serverfault|askubuntu)|([^\\.]+\\.stackexchange))\\.com$","wv":true,"al":false,"abd":true,"cpa_liid":[5882654614],"cpa_cid":[138377597667],"dp":false});var a=__webpack_require__(23),s=__webpack_require__(865),i=__webpack_require__(763);window.cam=new class{constructor(){this.gptImported=!1,this.collapsed={},e.cM("constructor"),this.clc_options=o,window.clcGamLoaderOptions?Object.assign(this,window.clcGamLoaderOptions):void 0===this.opt&&(this.opt=window.opt)}init(){if(e.cM("init"),void 0===this.opt)throw new Error("opt not set, required by GAM Loader");e.cM("setup message handler"),window.addEventListener("message",e=>{this.onmessage(e)}),this.opt.targeting&&this.opt.targeting.ProductVariant&&"SystemDefault"===this.opt.targeting.ProductVariant[0]&&(window.matchMedia&&window.matchMedia("(prefers-color-scheme: dark)").matches?this.opt.targeting.ProductVariant="Dark":this.opt.targeting.ProductVariant="Light")}handleSlotRenderedNoAdReport(){if(googletag.pubads().addEventListener("slotRenderEnded",e=>this.applyExtraMarginBottom(e)),Array.isArray(this.slotsRenderedEvents))for(var e=0;e<this.slotsRenderedEvents.length;e++)this.applyExtraMarginBottom(this.slotsRenderedEvents[e])}onmessage(t){let o="omni";if(t.data&&("string"==typeof t.data||t.data instanceof String))if(0===t.data.indexOf("get-omni-")){e.cM("Recevied get-omni message, sending back omni");var a=t.source,i=this.opt.omni,n="string"==typeof i?i:"";a.postMessage([o,n,this.opt.requestGuid].join("|"),"*")}else if(0===t.data.indexOf("collapse-")){e.cM("Recevied collapse message, collapse ad iframe"),e.cM(t);for(var r=t.source.window,l=document.getElementsByTagName("IFRAME"),d=0;d<l.length;d++){var c=l[d];if(c.contentWindow==r)return void(0,s.wo)(c.parentElement.parentElement.parentElement)}}else if(0===t.data.indexOf("resize|")){e.cM("Recevied resize message, resize ad iframe"),e.cM(t);let o=this._getFrameByEvent(t),a=t.data.indexOf("|")+1,s=t.data.slice(a),i=parseFloat(s)+.5;e.cM("New iframe height "+i),o.height=i.toString(),o.parentElement.style.height=i.toString()+"px"}else if(0===t.data.indexOf("getmarkup|")){let o=t.data.indexOf("|")+1,a=t.data.slice(o);e.cM("Recevied get markup message: "+a);let s=this._getFrameByEvent(t).closest(".everyonelovesstackoverflow");const i=document.createElement("script");i.dataset.adZoneId=s.id,i.src=a,document.body.appendChild(i)}}placeInIframe(t,o){e.cM("Received markup result in main frame");let a=document.getElementById(t.dataset.adZoneId),s=a.querySelector("iframe");if("string"==typeof o.prefixToIframeHtml){let e=document.createElement("div");e.innerHTML=o.prefixToIframeHtml,a.prepend(e.querySelector("img"))}s.contentWindow.postMessage(["markup",JSON.stringify(o)].join("|"),"*")}_getFrameByEvent(e){return Array.from(document.getElementsByTagName("iframe")).filter(t=>t.contentWindow===e.source)[0]}classifyZoneIds(e){const t=e.map(s.Nj).filter(s.yb);return{eligible:t.filter(s.xb).filter(s.pn),ineligible:t.filter(s.xj)}}applyExtraMarginBottom(t){if(t&&t.slot&&!t.isEmpty&&(t.creativeId||t.lineItemId||!t.isEmpty)){var o=t.slot.getSlotElementId();if(o){var s=document.getElementById(o);if(s)if((0,a.eq)(o)){var i=s?.closest(".js-zone-container");i.style.marginBottom="24px",e.cM("Applied extra margin to the bottom of "+o)}else e.cM("Not applying extra margin to the bottom of "+o+": shouldHaveReportButton = false");else e.cM("Not applying extra margin to the bottom of "+o+": resolved invalid adUnit element")}else e.cM("Not applying extra margin to the bottom of element: invalid adUnitElementId")}else e.cM("Not applying extra margin to the bottom of element: invalid SlotRenderEndedEvent")}load(o=(0,a.kG)()){const n=this.opt.tlb_position===t.Above?["dfp-mlb","dfp-smlb"]:["dfp-mlb","dfp-smlb","dfp-tlb"];if(!this.isGptReady())return e.cM("Initializing..."),this.initGpt(),void googletag.cmd.push(()=>this.load(o));this.opt.adReportEnabled?(e.cM("Ad reporting enabled"),this.adReports=new i.t(googletag,this)):(e.cM("Ad reporting not enabled"),this.handleSlotRenderedNoAdReport()),e.cM("Attempting to load ads into ids: ",o);const{eligible:r,ineligible:l}=this.classifyZoneIds(o);if(this.initDebugPanel(googletag,r.concat(l)),r.forEach(e=>(0,s.cf)(e)),l.forEach(s.wo),0===r.length)return void e.cM("Found no ad ids on page");e.cM("Eligible ids:",r),this.opt.abd&&this.appendAdblockDetector();var d=googletag.pubads().getSlots().filter(e=>o.indexOf(e.getSlotElementId())>=0);googletag.destroySlots(d),this.opt.sf&&(googletag.pubads().setForceSafeFrame(!0),googletag.pubads().setSafeFrameConfig({allowOverlayExpansion:!0,allowPushExpansion:!0,sandbox:!0})),e.cM("Targeting consent: Checking...");let c=!1,p=!1;void 0!==this.opt.targeting_consent&&(p=!0,e.cM("Targeting consent: Parameter set"),e.cM("Targeting consent: Consent given? ",this.opt.targeting_consent),c=this.opt.targeting_consent),void 0!==this.opt.personalization_consent&&(e.cM("Personalization consent: Parameter set"),e.cM("Personalization consent: Consent given? ",this.opt.personalization_consent),c=c&&this.opt.personalization_consent),c=c&&p,googletag.pubads().setRequestNonPersonalizedAds(c?0:1),c||googletag.pubads().setPrivacySettings({limitedAds:!0}),this.opt.ll||googletag.pubads().enableSingleRequest(),cam.sreEvent||(googletag.pubads().addEventListener("slotRenderEnded",e=>this.onSlotRendered(e)),cam.sreEvent=!0),this.setTargeting(googletag);var g=r.filter(e=>!this.opt.ll||n.indexOf(e.id)<0),h=r.filter(e=>!!this.opt.ll&&n.indexOf(e.id)>=0);e.cM("Up front ids:",g),e.cM("Lazy loaded ids:",h),g.forEach(t=>{e.cM(`Defining ad for element ${t.id}`),this.defineSlot(t.id,googletag),t.setAttribute("data-dfp-zone","true")}),googletag.enableServices(),g.forEach(t=>{e.cM(`Displaying ad for element ${t.id}`),googletag.cmd.push(()=>googletag.display(t.id))}),this.opt.ll&&(e.cM("Enabling lazy loading for GAM"),googletag.pubads().enableLazyLoad({fetchMarginPercent:0,renderMarginPercent:0}),e.cM("Setting up lazy loaded ad units"),h.forEach(t=>{e.cM(`Lazy loading - Defining Slot ${t.id}`),this.defineSlot(t.id,googletag)}),h.forEach(t=>{e.cM(`Lazy loading - Displaying ad for element ${t.id}`),googletag.cmd.push(()=>googletag.display(t.id))}))}setTargeting(t){let o=this.opt.targeting;if(!o)throw new Error("Targeting not defined");Object.keys(o).forEach(a=>{e.cM(`-> targeting - ${a}: ${o[a]}`),t.pubads().setTargeting(a,o[a])})}appendAdblockDetector(){const e=document.createElement("div");e.className="adsbox",e.id="clc-abd",e.style.position="absolute",e.style.pointerEvents="none",e.innerHTML="&nbsp;",document.body.appendChild(e)}onSlotRendered(o){try{const n=o.slot.getSlotElementId();let r=[];n||r.push("id=0");const l=document.getElementById(n);if(n&&!l&&r.push("el=0"),0!==r.length)return void this.stalled(r.join("&"));const{path:d,sizes:c,zone:p}=(0,a.Z7)(n);if(this.collapsed[p]&&o.isEmpty)return e.cM(`No line item for the element #${l.id}... collapsing.`),void(0,s.wo)(l);if(this.slotsRenderedEvents.push(o),o.lineItemId||o.creativeId||!o.isEmpty){e.cM(`Rendered ad for element #${l.id} [line item #${o.lineItemId}]`),e.cM(o);var i=l.parentElement;if(i.classList.contains("js-zone-container")){switch((0,s.cf)(i),n){case"dfp-tlb":this.opt.tlb_position===t.Above?i.classList.add("mb8"):i.classList.add("mt16");break;case"dfp-tag":i.classList.add("mb8");break;case"dfp-msb":i.classList.add("mt16");break;case"dfp-mlb":case"dfp-smlb":case"dfp-bmlb":i.classList.add("my8");break;case"dfp-isb":i.classList.add("mt24");break;case"dfp-m-aq":i.classList.add("my12"),i.classList.add("mx-auto")}(0,s.$Z)(i),(0,s.$Z)(l)}else e.cM(`No ad for element #${l.id}, collapsing`),e.cM(o),(0,s.wo)(l)}}catch(e){this.stalled("e=1")}}stalled(e){(new Image).src=`https://${this.clc_options.h}/stalled.gif?${e}`}defineSlot(t,o){"dfp-isb"===t&&(e.cM("-> targeting - Sidebar: Inline"),o.pubads().setTargeting("Sidebar",["Inline"])),"dfp-tsb"===t&&(e.cM("-> targeting - Sidebar: Right"),o.pubads().setTargeting("Sidebar",["Right"]));const{path:s,sizes:i,zone:n}=(0,a.Z7)(t);e.cM(`Defining slot for ${t}: ${s}, sizes: ${JSON.stringify(i)}`),o.defineSlot(s,i,t).addService(o.pubads())}importGptLibrary(){this.gptImported||(this.gptImported=!0,void 0===this.opt.targeting_consent||this.opt.targeting_consent?(0,s.Gx)("https://web.archive.org/web/20230306075114/https://securepubads.g.doubleclick.net/tag/js/gpt.js"):(0,s.Gx)("https://web.archive.org/web/20230306075114/https://pagead2.googlesyndication.com/tag/js/gpt.js"))}isGptReady(){return"undefined"!=typeof googletag&&!!googletag.apiReady}initGpt(){"undefined"==typeof googletag&&(window.googletag={cmd:(0,s.QZ)(()=>this.importGptLibrary())})}initDebugPanel(t,o){e.cM("initDebugPanel"),e.cM("Not showing debug panel")}},window.clcGamLoaderOptions&&(cam.init(),cam.load())})()})();</script> <footer id="footer" class="site-footer js-footer" role="contentinfo"> <div class="site-footer--container"> <nav class="site-footer--nav"> <div class="site-footer--col"> <h5 class="-title"><a href="/web/20230306075114/https://diy.stackexchange.com/">Home Improvement</a></h5> <ul class="-list js-primary-footer-links"> <li><a class="js-gps-track -link" data-gps-track="footer.click({ location: 2, link: 2 })" href="/web/20230306075114/https://diy.stackexchange.com/tour">Tour</a></li> <li><a href="/web/20230306075114/https://diy.stackexchange.com/help" class="js-gps-track -link" data-gps-track="footer.click({ location: 2, link: 3 })">Help</a></li> <li><a class="js-gps-track -link" data-gps-track="footer.click({ location: 2, link: 5 })" href="https://web.archive.org/web/20230306075114/https://chat.stackexchange.com/?tab=site&amp;host=diy.stackexchange.com">Chat</a></li> <li><a class="js-gps-track -link" data-gps-track="footer.click({ location: 2, link: 13 })" href="/web/20230306075114/https://diy.stackexchange.com/contact">Contact</a></li> <li><a class="js-gps-track -link" data-gps-track="footer.click({ location: 2, link: 14 })" href="https://web.archive.org/web/20230306075114/https://diy.meta.stackexchange.com/">Feedback</a></li> </ul> </div> <div class="site-footer--col"> <h5 class="-title"><a class="js-gps-track" data-gps-track="footer.click({ location: 2, link: 1 })" href="https://web.archive.org/web/20230306075114/https://stackoverflow.co/">Company</a></h5> <ul class="-list"> <li><a href="https://web.archive.org/web/20230306075114/https://stackoverflow.com/" class="js-gps-track -link" data-gps-track="footer.click({ location: 2, link: 15})">Stack Overflow</a></li> <li><a href="https://web.archive.org/web/20230306075114/https://stackoverflow.co/teams" class="js-gps-track -link" data-gps-track="footer.click({ location: 2, link: 29 })">Teams</a></li> <li><a href="https://web.archive.org/web/20230306075114/https://stackoverflow.co/advertising" class="js-gps-track -link" data-gps-track="footer.click({ location: 2, link: 21 })">Advertising</a></li> <li><a href="https://web.archive.org/web/20230306075114/https://stackoverflow.co/collectives" class="js-gps-track -link" data-gps-track="footer.click({ location: 2, link: 40 })">Collectives</a></li> <li><a href="https://web.archive.org/web/20230306075114/https://stackoverflow.co/talent" class="js-gps-track -link" data-gps-track="footer.click({ location: 2, link: 20 })">Talent</a></li> <li><a class="js-gps-track -link" data-gps-track="footer.click({ location: 2, link: 1 })" href="https://web.archive.org/web/20230306075114/https://stackoverflow.co/">About</a></li> <li><a class="js-gps-track -link" data-gps-track="footer.click({ location: 2, link: 27 })" href="https://web.archive.org/web/20230306075114/https://stackoverflow.co/company/press">Press</a></li> <li><a class="js-gps-track -link" data-gps-track="footer.click({ location: 2, link: 7 })" href="https://web.archive.org/web/20230306075114/https://stackoverflow.com/legal">Legal</a></li> <li><a class="js-gps-track -link" data-gps-track="footer.click({ location: 2, link: 8 })" href="https://web.archive.org/web/20230306075114/https://stackoverflow.com/legal/privacy-policy">Privacy Policy</a></li> <li><a class="js-gps-track -link" data-gps-track="footer.click({ location: 2, link: 37 })" href="https://web.archive.org/web/20230306075114/https://stackoverflow.com/legal/terms-of-service">Terms of Service</a></li> <li class="" id="consent-footer-link"><a class="js-gps-track -link js-cookie-settings" data-gps-track="footer.click({ location: 2, link: 38 })" href="#" data-consent-popup-loader="footer">Cookie Settings</a></li> <li><a class="js-gps-track -link" data-gps-track="footer.click({ location: 2, link: 39 })" href="https://web.archive.org/web/20230306075114/https://stackoverflow.com/legal/cookie-policy">Cookie Policy</a></li> </ul> </div> <div class="site-footer--col site-footer--categories-nav"> <div> <h5 class="-title"><a href="https://web.archive.org/web/20230306075114/https://stackexchange.com/" data-gps-track="footer.click({ location: 2, link: 30 })">Stack Exchange Network</a></h5> <ul class="-list"> <li> <a href="https://web.archive.org/web/20230306075114/https://stackexchange.com/sites#technology" class="-link js-gps-track" data-gps-track="footer.click({ location: 2, link: 24 })"> Technology </a> </li> <li> <a href="https://web.archive.org/web/20230306075114/https://stackexchange.com/sites#culturerecreation" class="-link js-gps-track" data-gps-track="footer.click({ location: 2, link: 24 })"> Culture &amp; recreation </a> </li> <li> <a href="https://web.archive.org/web/20230306075114/https://stackexchange.com/sites#lifearts" class="-link js-gps-track" data-gps-track="footer.click({ location: 2, link: 24 })"> Life &amp; arts </a> </li> <li> <a href="https://web.archive.org/web/20230306075114/https://stackexchange.com/sites#science" class="-link js-gps-track" data-gps-track="footer.click({ location: 2, link: 24 })"> Science </a> </li> <li> <a href="https://web.archive.org/web/20230306075114/https://stackexchange.com/sites#professional" class="-link js-gps-track" data-gps-track="footer.click({ location: 2, link: 24 })"> Professional </a> </li> <li> <a href="https://web.archive.org/web/20230306075114/https://stackexchange.com/sites#business" class="-link js-gps-track" data-gps-track="footer.click({ location: 2, link: 24 })"> Business </a> </li> <li class="mt16 md:mt0"> <a href="https://web.archive.org/web/20230306075114/https://api.stackexchange.com/" class="-link js-gps-track" data-gps-track="footer.click({ location: 2, link: 24 })"> API </a> </li> <li> <a href="https://web.archive.org/web/20230306075114/https://data.stackexchange.com/" class="-link js-gps-track" data-gps-track="footer.click({ location: 2, link: 24 })"> Data </a> </li> </ul> </div> </div> </nav> <div class="site-footer--copyright fs-fine md:mt24"> <ul class="-list -social md:mb8"> <li><a class="js-gps-track -link" data-gps-track="footer.click({ location: 2, link:4 })" href="https://web.archive.org/web/20230306075114/https://stackoverflow.blog/?blb=1">Blog</a></li> <li><a href="https://web.archive.org/web/20230306075114/https://www.facebook.com/officialstackoverflow/" class="-link js-gps-track" data-gps-track="footer.click({ location: 2, link: 31 })">Facebook</a></li> <li><a href="https://web.archive.org/web/20230306075114/https://twitter.com/stackoverflow" class="-link js-gps-track" data-gps-track="footer.click({ location: 2, link: 32 })">Twitter</a></li> <li><a href="https://web.archive.org/web/20230306075114/https://linkedin.com/company/stack-overflow" class="-link js-gps-track" data-gps-track="footer.click({ location: 2, link: 33 })">LinkedIn</a></li> <li><a href="https://web.archive.org/web/20230306075114/https://www.instagram.com/thestackoverflow" class="-link js-gps-track" data-gps-track="footer.click({ location: 2, link: 36 })">Instagram</a></li> </ul> <p class="md:mb0"> Site design / logo &#169; 2023 Stack Exchange Inc; user contributions licensed under <span class="td-underline"><a href="https://web.archive.org/web/20230306075114/https://stackoverflow.com/help/licensing">CC BY-SA</a></span>. <span id="svnrev">rev&nbsp;2023.3.3.43278</span> </p> </div> </div> </footer> <!-- Google tag (gtag.js) --> <script async src="https://web.archive.org/web/20230306075114js_/https://www.googletagmanager.com/gtag/js?id=G-S812YQPLT2"></script> <script> window.dataLayer = window.dataLayer || []; function gtag() { dataLayer.push(arguments); } </script> <script> StackExchange.ready(function() { var ga3Settings = { autoLink: ["stackoverflow.blog","info.stackoverflowsolutions.com","stackoverflowsolutions.com"], sendTitles: true, tracker: window.ga, trackingCodes: [ 'UA-108242619-5' ], checkDimension: 'dimension42' }; var customGA4Dimensions = {}; var ga4Settings = { tracker: gtag, trackingCodes: [ 'G-S812YQPLT2' ], consentsToPerformanceCookies: "denied", consentsToTargetingCookies: "denied", eventParameters: customGA4Dimensions, checkForAdBlock: true }; StackExchange.ga.init({ GA3: ga3Settings, GA4: ga4Settings }); StackExchange.ga.setDimension('dimension2', '|repair|walls|'); StackExchange.ga.setDimension('dimension3', 'Questions/Show'); StackExchange.ga.setDimension('dimension7', "1678089074.632690448"); StackExchange.ga.trackPageView(); }); </script> <div class="ff-sans ps-fixed z-nav-fixed ws4 sm:w-auto p32 sm:p16 bg-black-750 fc-white bar-lg b16 l16 r16 js-consent-banner"> <svg aria-hidden="true" class="mln4 mb24 sm:d-none svg-spot spotCookieLg" style="color: var(--theme-button-filled-background-color)" width="96" height="96" viewbox="0 0 96 96"> <path d="M35 45.5a7.5 7.5 0 11-15 0 7.5 7.5 0 0115 0zM63.5 63a7.5 7.5 0 100-15 7.5 7.5 0 000 15zm-19 19a7.5 7.5 0 100-15 7.5 7.5 0 000 15z" opacity=".2"/> <path d="M56.99 2.53a23.1 23.1 0 0114.66 6.15h.01l.01.02c.57.55.61 1.27.5 1.74v.07a10.95 10.95 0 01-3.07 4.77 9 9 0 01-6.9 2.5 10.34 10.34 0 01-9.72-10.44v-.08a10 10 0 011.03-3.74l.01-.03.02-.02c.28-.5.82-.92 1.52-.95.63-.02 1.27-.02 1.93.01zm12.04 7.83a20.1 20.1 0 00-12.2-4.83l-.92-.03c-.23.6-.38 1.25-.43 1.94a7.34 7.34 0 006.95 7.34 6 6 0 004.64-1.7c.94-.88 1.6-1.9 1.96-2.72zm15.3 8.76a6.84 6.84 0 00-5.09-.24 7.9 7.9 0 00-3.28 2.05 1.8 1.8 0 00-.3 1.95l.02.02v.02a15.16 15.16 0 008.74 7.47c.64.23 1.32.08 1.8-.33a6.63 6.63 0 001.63-1.97l.01-.03.01-.03c1.67-3.5-.12-7.32-3.54-8.91zm-5.5 3.28c.36-.25.82-.5 1.35-.67.92-.3 1.92-.35 2.89.1 2.14 1 2.92 3.14 2.11 4.88-.12.21-.26.41-.43.6l-.26-.1a12.29 12.29 0 01-5.66-4.81zM32 24a2 2 0 11-4 0 2 2 0 014 0zm12 21a2 2 0 11-4 0 2 2 0 014 0zm36 4a2 2 0 11-4 0 2 2 0 014 0zm-7 21a2 2 0 11-4 0 2 2 0 014 0zM59 81a2 2 0 11-4 0 2 2 0 014 0zM22 63a2 2 0 11-4 0 2 2 0 014 0zm27 7a9 9 0 11-18 0 9 9 0 0118 0zm-3 0a6 6 0 10-12 0 6 6 0 0012 0zM33 41a9 9 0 11-18 0 9 9 0 0118 0zm-15 0a6 6 0 1012 0 6 6 0 00-12 0zm50 11a9 9 0 11-18 0 9 9 0 0118 0zm-3 0a6 6 0 10-12 0 6 6 0 0012 0zM44.08 4.24c.31.48.33 1.09.05 1.58a17.46 17.46 0 00-2.36 8.8c0 9.55 7.58 17.24 16.85 17.24 2.97 0 5.75-.78 8.16-2.15a1.5 1.5 0 012.1.66 12.08 12.08 0 0011 6.74 12.4 12.4 0 007.85-2.75 1.5 1.5 0 012.38.74A45.76 45.76 0 0192 48.16c0 24.77-19.67 44.9-44 44.9S4 72.93 4 48.16C4 25.23 20.84 6.28 42.64 3.58a1.5 1.5 0 011.44.66zM40.22 7C21.32 10.71 7 27.7 7 48.16c0 23.17 18.39 41.9 41 41.9s41-18.73 41-41.9c0-3.52-.42-6.93-1.22-10.2a15.5 15.5 0 01-7.9 2.15c-5.5 0-10.36-2.83-12.97-7.1a19.46 19.46 0 01-8.28 1.85c-11 0-19.86-9.1-19.86-20.24 0-2.7.52-5.26 1.45-7.62zM92 91a2 2 0 100-4 2 2 0 000 4zM7 8.5a2.5 2.5 0 11-5 0 2.5 2.5 0 015 0zM82.5 90a1.5 1.5 0 100-3 1.5 1.5 0 000 3zm9.5-7.5a1.5 1.5 0 11-3 0 1.5 1.5 0 013 0zM13.5 8a1.5 1.5 0 100-3 1.5 1.5 0 000 3zM80 14.5a1.5 1.5 0 11-3 0 1.5 1.5 0 013 0zM53.5 20a1.5 1.5 0 100-3 1.5 1.5 0 000 3z"/> </svg> <p class="fs-body2 fw-bold mb4"> Your privacy </p> <p class="mb16 s-anchors s-anchors__inherit s-anchors__underlined"> By clicking “Accept all cookies”, you agree Stack Exchange can store cookies on your device and disclose information in accordance with our <a href="https://web.archive.org/web/20230306075114/https://stackoverflow.com/legal/cookie-policy">Cookie Policy</a>. </p> <div class="d-flex gs8 sm:fd-column"> <button class="flex--item6 s-btn s-btn__primary js-accept-cookies js-consent-banner-hide"> Accept all cookies </button> <button class="flex--item6 s-btn s-btn__primary js-reject-cookies js-consent-banner-hide"> Necessary cookies only </button> </div> <div class="d-flex mt8 sm:fd-column"> <button class="flex--item12 s-btn s-btn__filled js-cookie-settings" data-consent-popup-loader="banner"> Customize settings </button> </div> </div> <div id="onetrust-consent-sdk" class="d-none"></div> <div id="onetrust-banner-sdk" data-controller="s-modal"></div> <div id="ot-pc-content" class="d-none"></div> <div id="onetrust-style" class="d-none">&nbsp;</div> <div class="d-none js-consent-banner-version" data-consent-banner-version="a1"></div> </body> </html> <!-- FILE ARCHIVED ON 07:51:14 Mar 06, 2023 AND RETRIEVED FROM THE INTERNET ARCHIVE ON 06:33:55 Nov 30, 2024. JAVASCRIPT APPENDED BY WAYBACK MACHINE, COPYRIGHT INTERNET ARCHIVE. ALL OTHER CONTENT MAY ALSO BE PROTECTED BY COPYRIGHT (17 U.S.C. SECTION 108(a)(3)). --> <!-- playback timings (ms): captures_list: 1.015 exclusion.robots: 0.062 exclusion.robots.policy: 0.046 esindex: 0.014 cdx.remote: 309.839 LoadShardBlock: 181.828 (3) PetaboxLoader3.datanode: 145.159 (4) load_resource: 164.803 PetaboxLoader3.resolve: 112.348 -->

Pages: 1 2 3 4 5 6 7 8 9 10